FGI1: Abgaben hinzugefügt

This commit is contained in:
Jim Martens 2014-12-10 23:04:56 +01:00
parent cd9612c504
commit b3e2fbeefc
12 changed files with 1522 additions and 0 deletions

148
fgi1/Aufgabenblatt10.tex Normal file
View File

@ -0,0 +1,148 @@
\documentclass[10pt,a4paper,oneside,ngerman,numbers=noenddot]{scrartcl}
\usepackage[T1]{fontenc}
\usepackage[utf8]{inputenc}
\usepackage[ngerman]{babel}
\usepackage{amsmath}
\usepackage{amsfonts}
\usepackage{amssymb}
\usepackage{paralist}
\usepackage{gauss}
\usepackage{stmaryrd}
\usepackage[locale=DE,exponent-product=\cdot,detect-all]{siunitx}
\usepackage{tikz}
\usetikzlibrary{automata,matrix,fadings,calc,positioning,decorations.pathreplacing,arrows,decorations.markings}
\usepackage{polynom}
\polyset{style=C, div=:,vars=x}
\pagenumbering{arabic}
\def\thesection{10.\arabic{section})}
\def\thesubsection{\arabic{subsection}.}
\def\thesubsubsection{(\alph{subsubsection})}
\setcounter{section}{2}
\makeatletter
\renewcommand*\env@matrix[1][*\c@MaxMatrixCols c]{%
\hskip -\arraycolsep
\let\@ifnextchar\new@ifnextchar
\array{#1}}
\makeatother
\addtolength{\parskip}{\baselineskip}
\begin{document}
\author{Jim Martens}
\title{Hausaufgaben zum 11. Juni}
\maketitle
\section{} %10.3
\subsection{} %1.
\subsubsection{} %a
Es ist zu zeigen, dass F aus der angegebenen Formelmenge M folgt. Nach Definition 5.1 folgt F genau dann aus M, wenn alle Modelle von M auch Modelle von F sind. Nach Satz 5.11 sind die Modelle einer Formelmenge identisch mit den Modellen einer Konjunktion aller Mengenglieder.\\
Daraus ergibt sich, dass eine Belegung diese Formel erfüllen muss, um ein Modell von M zu sein:\\
$G = ((B \Rightarrow D) \Rightarrow (A \vee C)) \wedge ((A \vee C) \Rightarrow E) \wedge (B \Rightarrow (D \vee E)) \wedge (E \Rightarrow F)$\\
Daraus folgt wiederum, dass eine Belegung M nur erfüllt, wenn $(E \Rightarrow F)$ erfüllt ist. Diese Teilformel ist wiederum erfüllt, wenn E falsch oder F wahr ist.
M kann nur erfüllt sein, wenn E wahr ist, denn sonst müsste auch $(A \vee C)$ und damit $(B \Rightarrow D)$ falsch sein. Letzteres würde voraussetzen, dass B wahr und D falsch sind. Daraus würde folgen, dass $(B \Rightarrow (D \vee E))$ falsch ergäbe, da hier nun B wahr und sowohl D als auch E falsch wären. Damit wiederum wäre die Formel G nicht erfüllt.\\
Daraus folgt also, dass E und damit auch F wahr sein müssen, damit eine Belegung ein Modell von M sein kann. Daher ist immer wahr, wenn eine Belegung ein Modell von M ist, womit diese Belegung auch ein Modell von F ist.
\subsubsection{} %b
In diesem Fall kann E falsch sein. Damit müssten auch $(A \vee C)$ und $\neg(B \Rightarrow D)$ falsch sein. $(B \Rightarrow D)$ wiederum müsste wahr sein, was durch den Wahrheitswert falsch für B erreicht werden kann. Damit kann eine Belegung M erfüllen, wenn E falsch ist. Wenn E falsch ist, dann muss F nicht wahr sein, damit $(E \Rightarrow F)$ wahr ist, womit nicht alle Modelle von M auch Modelle von F sind.
\subsection{} %2.
F folgt genau dann aus M, wenn jede Belegung, die M wahr macht, auch F wahr macht. Damit folgt F genau dann aus M, wenn jede Belegung, die alle Formeln aus M wahr macht, $\neg F$ falsch macht. Daher kann F nicht aus $M \cup \{\neg F\}$ folgen, da jede Belegung, die diese Menge erfüllt, F falsifiziert. Wenn F nicht aus $M \cup \{\neg F\}$, dann darf F auch nicht aus M folgen. Dies ist nur möglich, wenn M unerfüllbar ist.
\section{} %10.4
\subsection{} %1.
\setcounter{subsubsection}{0}
\subsubsection{} %a
\begin{alignat*}{2}
\text{sub}_{1a}(A) &=& (A \Rightarrow \neg B) \\
\text{sub}_{1a}(B) &=& (D \wedge A) \\
\text{sub}_{1a}(C) &=& (C \vee D) \\
\intertext{Für alle anderen Aussagensymbole $A_{i}$ sei $sub_{1a}(A_{i}) = A_{i}$}
sub_{1a}(F_{a}) &=& (((A \Rightarrow \neg B) \Rightarrow \neg(D \wedge A)) \wedge (C \vee D)) = G_{a}
\end{alignat*}
\subsubsection{} %b
\begin{alignat*}{2}
\text{sub}_{1b}(A) &=& (\neg(B \vee C) \wedge E) \\
\text{sub}_{1b}(D) &=& \neg(B \vee C) \\
\intertext{Für alle anderen Aussagensymbole $A_{i}$ sei $sub_{1b}(A_{i}) = A_{i}$}
sub_{1b}(F_{b}) &=& ((\neg(B \vee C) \wedge E) \Leftrightarrow \neg(B \vee C)) \\
&=& ((\neg(B \vee C) \wedge E) \Leftrightarrow \neg(B \vee C)) = sub_{1b}(G_{b})
\end{alignat*}
\subsection{} %2.
\subsubsection{} %a
$R_{a} = \frac{\neg A}{A \Rightarrow B}$\\
\begin{tabular}{c|cc|cc}
& F & G & $\neg$F & $(F \Rightarrow G)$ \\
\hline
$\mathcal{A}_{0}$ & 0 & 0 & 1 & 1 \\
$\mathcal{A}_{1}$ & 0 & 1 & 1 & 1 \\
$\mathcal{A}_{2}$ & 1 & 0 & 0 & 0 \\
$\mathcal{A}_{3}$ & 1 & 1 & 0 & 1
\end{tabular}\\
Für alle Belegungen mit $\mathcal{A}(\neg F) = 1$ gilt auch $\mathcal{A}((F \Rightarrow G)) = 1$.\\
Falls M eine Formelmenge ist und $M \vdash_{R_{a}} G$, dann $M \models G$. \\
Daher ist die Regel korrekt.
\subsubsection{} %b
$R_{b} = \frac{A \Leftrightarrow B}{A}$\\
\begin{tabular}{c|cc|c}
& F & G & $(F \Leftrightarrow G)$ \\
\hline
$\mathcal{A}_{0}$ & 0 & 0 & 1 \\
$\mathcal{A}_{1}$ & 0 & 1 & 0 \\
$\mathcal{A}_{2}$ & 1 & 0 & 0 \\
$\mathcal{A}_{3}$ & 1 & 1 & 1
\end{tabular}\\
Für die Belegung $\mathcal{A}_{0}$ mit $\mathcal{A}((F \Leftrightarrow G)) = 1$ gilt nicht $\mathcal{A}(F) = 1$. Demzufolge ist nicht jedes Modell von $(F \Leftrightarrow G)$ auch eines von $F$. Daher ist die Regel nicht korrekt.
\subsubsection{} %c
$R_{c} = \frac{A \vee B, B \vee C}{A \vee C}$\\
\begin{tabular}{c|ccc|ccc}
& F & G & H & $(F \vee G)$ & $(G \vee H)$ & $(F \vee H)$ \\
\hline
$\mathcal{A}_{0}$ & 0 & 0 & 0 & 0 & 0 & 0\\
$\mathcal{A}_{1}$ & 0 & 0 & 1 & 0 & 1 & 1\\
$\mathcal{A}_{2}$ & 0 & 1 & 0 & 1 & 1 & 0\\
$\mathcal{A}_{3}$ & 0 & 1 & 1 & 1 & 1 & 1\\
$\mathcal{A}_{4}$ & 1 & 0 & 0 & 1 & 0 & 1\\
$\mathcal{A}_{5}$ & 1 & 0 & 1 & 1 & 1 & 1\\
$\mathcal{A}_{6}$ & 1 & 1 & 0 & 1 & 1 & 1\\
$\mathcal{A}_{7}$ & 1 & 1 & 1 & 1 & 1 & 1\\
\end{tabular}\\
Für die Belegung $\mathcal{A}_{2}$ mit $\mathcal{A}((F \vee G)) = 1$ und $\mathcal{A}((G \vee H)) = 1$ gilt nicht $\mathcal{A}((F \vee H)) = 1$.\\
Demzufolge erfüllt nicht jede Belegung, die $(F \vee G)$ und $(G \vee H)$ erfüllt, auch $(F \vee H)$. Daher ist die Regel nicht korrekt.
\subsection{} %3.
\def\thesubsubsection{(\arabic{subsubsection})}
\subsubsection{} %1
Annahme aus M
\subsubsection{} %2
Axiom $H_{6b}$ \\
$sub_{2}(A) = (G \Rightarrow H)$ \\
$sub_{2}(B) = (H \Rightarrow F)$
\subsubsection{} %3
Modus ponens\\
$sub_{3}(A) = (G \Rightarrow H) \wedge (H \Rightarrow F)$ \\
$sub_{3}(B) = (H \Rightarrow F)$
\subsubsection{} %4
Axiom $H_{1}$ \\
$sub_{4}(A) = (H \Rightarrow F)$\\
$sub_{4}(B) = G$
\subsubsection{} %5
Modus ponens\\
$sub_{5}(A) = (H \Rightarrow F)$\\
$sub_{5}(B) = (G \Rightarrow (H \Rightarrow F))$
\subsubsection{} %6
Axiom $H_{6a}$\\
$sub_{6}(A) = (G \Rightarrow H)$\\
$sub_{6}(B) = (H \Rightarrow F)$
\subsubsection{} %7
Modus ponens\\
$sub_{7}(A) = (G \Rightarrow (H \Rightarrow F))$ \\
$sub_{7}(B) = (G \Rightarrow H)$
\subsubsection{} %8
Axiom $H_{2}$\\
$sub_{8}(A) = G$\\
$sub_{8}(B) = H$\\
$sub_{8}(C) = F$
\subsubsection{} %9
Modus ponens\\
$sub_{9}(A) = (G \Rightarrow H)$\\
$sub_{9}(B) = ((G \Rightarrow (H \Rightarrow F)) \Rightarrow (G \Rightarrow F))$
\subsubsection{} %10
Modus ponens\\
$sub_{10}(A) = (G \Rightarrow (H \Rightarrow F))$\\
$sub_{10}(B) = (G \Rightarrow F)$
\end{document}

171
fgi1/Aufgabenblatt11.tex Normal file
View File

@ -0,0 +1,171 @@
\documentclass[10pt,a4paper,oneside,ngerman,numbers=noenddot]{scrartcl}
\usepackage[T1]{fontenc}
\usepackage[utf8]{inputenc}
\usepackage[ngerman]{babel}
\usepackage{amsmath}
\usepackage{amsfonts}
\usepackage{amssymb}
\usepackage{paralist}
\usepackage{gauss}
\usepackage{stmaryrd}
\usepackage[locale=DE,exponent-product=\cdot,detect-all]{siunitx}
\usepackage{tikz}
\usetikzlibrary{automata,matrix,fadings,calc,positioning,decorations.pathreplacing,arrows,decorations.markings}
\usepackage{polynom}
\polyset{style=C, div=:,vars=x}
\pagenumbering{arabic}
\def\thesection{11.\arabic{section})}
\def\thesubsection{\arabic{subsection}.}
\def\thesubsubsection{(\alph{subsubsection})}
\setcounter{section}{2}
\makeatletter
\renewcommand*\env@matrix[1][*\c@MaxMatrixCols c]{%
\hskip -\arraycolsep
\let\@ifnextchar\new@ifnextchar
\array{#1}}
\makeatother
\addtolength{\parskip}{\baselineskip}
\begin{document}
\author{Jim Martens}
\title{Hausaufgaben zum 25. Juni}
\maketitle
\section{} %11.3
\subsection{} %1.
\subsubsection{} %a
$M = \{\{A\}\}$\\
\begin{tabular}{c|cc}
& A \\
\hline
$\mathcal{A}_{0}$ & 0 \\
$\mathcal{A}_{1}$ & 1
\end{tabular}
\subsubsection{} %b
$M = \{\{A,B,C\}\}$\\
\begin{tabular}{c|ccc|ccc}
& A & B & C & $\{A,B,C\}$ & $\{\{A,B,C\}\}$ & $A \vee (B \vee C)$\\
\hline
$\mathcal{A}_{0}$ & 0 & 0 & 0 & 0 & 0 & 0 \\
$\mathcal{A}_{1}$ & 0 & 0 & 1 & 1 & 1 & 1 \\
$\mathcal{A}_{2}$ & 0 & 1 & 0 & 1 & 1 & 1\\
$\mathcal{A}_{3}$ & 0 & 1 & 1 & 1 & 1 & 1\\
$\mathcal{A}_{4}$ & 1 & 0 & 0 & 1 & 1 & 1\\
$\mathcal{A}_{5}$ & 1 & 0 & 1 & 1 & 1 & 1\\
$\mathcal{A}_{6}$ & 1 & 1 & 0 & 1 & 1 & 1\\
$\mathcal{A}_{7}$ & 1 & 1 & 1 & 1 & 1 & 1\\
\end{tabular}
\subsubsection{} %c
$M = \{\{A\}, \{B\}, \{C\}\}$\\
\begin{tabular}{c|ccc|ccc|cc}
& A & B & C & $\{A\}$ & $\{B\}$ & $\{C\}$ & $\{\{A\}, \{B\}, \{C\}\}$ & $A \wedge (B \wedge C)$\\
\hline
$\mathcal{A}_{0}$ & 0 & 0 & 0 & 0 & 0 & 0 & 0 & 0 \\
$\mathcal{A}_{1}$ & 0 & 0 & 1 & 0 & 0 & 1 & 0 & 0 \\
$\mathcal{A}_{2}$ & 0 & 1 & 0 & 0 & 1 & 0 & 0 & 0\\
$\mathcal{A}_{3}$ & 0 & 1 & 1 & 0 & 1 & 1 & 0 & 0\\
$\mathcal{A}_{4}$ & 1 & 0 & 0 & 1 & 0 & 0 & 0 & 0\\
$\mathcal{A}_{5}$ & 1 & 0 & 1 & 1 & 0 & 1 & 0 & 0\\
$\mathcal{A}_{6}$ & 1 & 1 & 0 & 1 & 1 & 0 & 0 & 0\\
$\mathcal{A}_{7}$ & 1 & 1 & 1 & 1 & 1 & 1 & 1 & 1\\
\end{tabular}
\subsection{} %2.
\begin{alignat*}{2}
M_{1} &= \{\{A\}\} \\
M_{2} &= \{\{\neg A\}\} \\
M_{3} &= \{\{A, \neg A\}\} \\
M_{4} &= \{\{A\}, \{\neg A\}\} \\
M_{5} &= \{\{A\}, \{A, \neg A\}\} \\
M_{6} &= \{\{\neg A\}, \{A, \neg A\}\} \\
M_{7} &= \{\{A\}, \{\neg A\}, \{A, \neg A\}\}
\end{alignat*}\\
$M_{3}$ ist allgemeingültig. $M_{4}$ und $M_{7}$ sind unerfüllbar. $M_{1}$ und $M_{5}$, sowie $M_{2}$ und $M_{6}$ sind äquivalent zueinander.
%\begin{tabular}{c|c|ccccccc}
%& A & $\{\{A\}\}$ & $\{\{\neg A\}\}$ & $\{\{A, \neg A\}\}$ & $\{\{A\}, \{\neg A\}\}$ & $\{\{A\}, \{A, \neg A\}\}$ & $\{\{\neg A\}, \{A, \neg A\}\}$ & $\{\{A\}, \{\neg A\}, \{A, \neg A\}\}$\\
%\hline
%$\mathcal{A}_{0}$ & 0 & 0 & 1 & 1 & 0 & 0 & 1 & 0 \\
%$\mathcal{A}_{1}$ & 1 & 1 & 0 & 1 & 0 & 1 & 0 & 0
%\end{tabular}
\subsection{} %3.
Behauptung: $M \cup \{K\}$ ist genau dann erfüllbar, wenn M erfüllbar ist.\\
Beweis:\\
Es ist zu zeigen, dass (1) $M \cup \{K\}$ erfüllbar ist, wenn M erfüllbar ist und dass (2) M erfüllbar ist, wenn $M \cup \{K\}$ erfüllbar ist.\\
\\
Teilbeweis von (1):\\
M sei erfüllbar.\\
$M \cup \{K\}$ ist erfüllbar, wenn es mindestens eine Belegung gibt, für die sowohl M als auch K erfüllt sind.\\
K ist erfüllt, wenn mindestens eines der Literale in K erfüllt ist.\\
\\
Es gilt $A, \neg A \in K$. Unabhängig von der Belegung von A ist K daher immer erfüllt, denn für $\mathcal{A}(A) = 0$ gilt $\mathcal{A}(\neg A) = 1$ und für $\mathcal{A}(A) = 1$ gilt $\mathcal{A}(A) = 1$.\\
Da K immer erfüllt ist, stellt es bei der Vereinigung mit M keine einschränkende Bedingung dar. Ob die vereinigte Menge erfüllbar ist, hängt damit von M ab. Da M aufgrund der Annahme erfüllbar ist, ist somit auch $M \cup \{K\}$ erfüllbar.\\
\\
Teilbeweis für (2):\\
$M \cup \{K\}$ sei erfüllbar.\\
$M \cup \{K\}$ ist erfüllbar, wenn es mindestens eine Belegung gibt, für die sowohl M als auch K erfüllt sind. Wie bereits gezeigt, ist K für jede Belegung erfüllt. Damit stellt K keine zusätzliche Bedingung und $M \cup \{K\}$ ist erfüllbar, wenn M erfüllbar ist.\\
Aufgrund der Annahme ist $M \cup \{K\}$ erfüllbar, womit M ebenfalls erfüllbar ist.\\
\\
Es konnten beide Richtungen gezeigt werden, womit die Behauptung bewiesen ist.
\section{} %11.4
\subsection{} %1.
\setcounter{subsubsection}{0}
\subsubsection{} %a
\begin{alignat*}{2}
F_{3} &=& C \wedge (\neg A \vee \neg E \vee D) \wedge E \wedge (\neg C \vee B) \wedge (\neg E \vee \neg B \vee A) \wedge (\neg D \vee \neg C \vee \neg A) \\
\intertext{Umformen in Implikationsschreibweise}
&=& (T \Rightarrow C) \wedge ((A \wedge E) \Rightarrow D) \wedge (T \Rightarrow E) \wedge (C \Rightarrow B) \wedge ((E \wedge B) \Rightarrow A) \wedge ((D \wedge C \wedge A) \Rightarrow \perp) \\
\intertext{Anwenden des Markierungsalgorithmus}
&=& (T \Rightarrow C^{1}) \wedge ((A^{3} \wedge E^{1}) \Rightarrow D^{4}) \wedge (T \Rightarrow E^{1}) \wedge (C^{1} \Rightarrow B^{2}) \wedge ((E^{1} \wedge B^{2}) \Rightarrow A^{3}) \wedge ((D^{4} \wedge C^{1} \wedge A^{3}) \Rightarrow \perp) \\
\intertext{Die Formel ist unerfüllbar.}
\end{alignat*}\\
\\\\\\
Anwenden der P-Resolution:\\
\begin{tabular}{cccccc}
($\neg D \vee \neg C \vee \neg A$) & C & ($\neg C \vee B$) & ($\neg E \vee \neg B \vee A$) & E & ($\neg A \vee \neg E \vee D$) \\
& ($\neg D \vee \neg A$) & B & ($\neg B \vee A$) & & ($\neg A \vee D$) \\
& & & A & & \\
& & $\neg D$ & & D & \\
& & & $\Box$ & &
\end{tabular} \\
Die Formel ist auch nach der P-Resolution unerfüllbar.
\subsubsection{} %b
\begin{alignat*}{2}
F_{4} &=& \neg A \wedge (\neg A \vee C \vee \neg E) \wedge (A \vee \neg C \vee \neg E) \wedge B \wedge (\neg B \vee D) \wedge (\neg B \vee \neg D \vee E) \\
\intertext{Umformen in Implikationsschreibweise}
&=& (A \Rightarrow \perp) \wedge ((A \wedge E) \Rightarrow C) \wedge ((C \wedge E) \Rightarrow A) \wedge (T \Rightarrow B) \wedge (B \Rightarrow D) \wedge ((B \wedge D) \Rightarrow E) \\
\intertext{Anwenden des Markierungsalgorithmus}
&=& (A \Rightarrow \perp) \wedge ((A \wedge E^{3}) \Rightarrow C) \wedge ((C \wedge E^{3}) \Rightarrow A) \wedge (T \Rightarrow B^{1}) \wedge (B^{1} \Rightarrow D^{2}) \wedge ((B^{1} \wedge D^{2}) \Rightarrow E^{3}) \\
\intertext{Die Formel ist erfüllbar.}
\end{alignat*}\\
Anwenden der P-Resolution:\\
\begin{tabular}{cccccc}
($\neg A \vee C \vee \neg E$) & ($\neg B \vee D$) & B & ($\neg B \vee \neg D \vee E$) & ($A \vee \neg C \vee \neg E$) \\
& D & & ($\neg D \vee E$) & & \\
& & E & & & \\
& ($\neg A \vee C$) & & ($A \vee \neg C$) & &
\end{tabular}\\
Es können keine weiteren Resolventen mit der P-Resolution gebildet werden. Die Formel ist daher nicht unerfüllbar und damit erfüllbar.
\subsection{} %2.
\begin{alignat*}{2}
F_{5} &=& (A \vee B) \wedge (A \vee D) \wedge (C \vee D) \wedge (\neg A \vee \neg C) \wedge (\neg B \vee \neg D) \wedge (B \vee \neg A \vee \neg D)
\end{alignat*}\\
Anwenden der P-Resolution:\\
\begin{tabular}{cccccc}
\{A,B\} & \{$\neg B, \neg D$\} & \{A,D\} & \{$\neg A, \neg C$\} & \{C,D\} & \{$B, \neg A, \neg D$\} \\
& & \{$A, \neg B$\} & & & \\
& \{A\} & & & & \\
& & \{$\neg C$\} & & \{$B, \neg D$\} & \\
& & & \{D\} & & \\
& \{$\neg B$\} & & & \{B\} & \\
& & & $\Box$ & &
\end{tabular}\\
Die Formel ist nach der P-Resolution unerfüllbar.
\subsection{} %3.
\begin{alignat*}{2}
F_{6} &=& (A \Rightarrow C) \Rightarrow ((B \Rightarrow C) \Rightarrow ((A \vee B) \Rightarrow C)) \\
\intertext{Eliminieren von Implikationen}
&=& \neg(\neg A \vee C) \vee (\neg(\neg B \vee C) \vee (\neg(A \vee B) \vee C)) \\
\intertext{Negationen nach innen ziehen}
&=& (A \wedge \neg C) \vee ((B \wedge \neg C) \vee ((\neg A \wedge \neg B) \vee C))
\end{alignat*}\\
\end{document}

106
fgi1/Aufgabenblatt12.tex Normal file
View File

@ -0,0 +1,106 @@
\documentclass[10pt,a4paper,oneside,ngerman,numbers=noenddot]{scrartcl}
\usepackage[T1]{fontenc}
\usepackage[utf8]{inputenc}
\usepackage[ngerman]{babel}
\usepackage{amsmath}
\usepackage{amsfonts}
\usepackage{amssymb}
\usepackage{paralist}
\usepackage{gauss}
\usepackage{stmaryrd}
\usepackage[locale=DE,exponent-product=\cdot,detect-all]{siunitx}
\usepackage{tikz}
\usetikzlibrary{automata,matrix,fadings,calc,positioning,decorations.pathreplacing,arrows,decorations.markings}
\usepackage{polynom}
\polyset{style=C, div=:,vars=x}
\pagenumbering{arabic}
\def\thesection{12.\arabic{section})}
\def\thesubsection{\arabic{subsection}.}
\def\thesubsubsection{(\alph{subsubsection})}
\setcounter{section}{2}
\makeatletter
\renewcommand*\env@matrix[1][*\c@MaxMatrixCols c]{%
\hskip -\arraycolsep
\let\@ifnextchar\new@ifnextchar
\array{#1}}
\makeatother
\addtolength{\parskip}{\baselineskip}
\begin{document}
\author{Jim Martens}
\title{Hausaufgaben zum 2. Juli}
\maketitle
\section{} %12.3
\subsection{} %1.
\begin{itemize}
\item Q und t müssen Variablen sein, da sie hinter Quantoren auftreten.
\item z(t), m(S, Q) und m(Q(S), z) müssen Formeln sein, da sie mit Junktoren verknüpft werden.
\item Entsprechend müssen z und m Prädikatensymbole sein und S, Q(S) und z Terme. z ist einstellig und m ist zweistellig.
\item S und z könnten eine Variable oder eine Konstante sein, denn sie sind atomare Terme.
\item Q ist ein Funktionssymbol und einstellig.
\end{itemize}
Das Problem der Formalisierung ist, dass Q eine Variable sein muss und gleichzeitig ein Funktionssymbol ist. Ebenso ist z einmal ein Prädikatensymbol und einmal ein atomarer Term.
\subsection{} %2.
\begin{tikzpicture}[shorten >=1pt,node distance=1.1cm,on grid]
\node (A_z) {$\forall z$};
\node (Impl) [below=of A_z]{$\Rightarrow$};
\node (E_x) [below left=1.0 and 2.0 of Impl] {$\exists x$};
\node (E_y) [below right=1.0 and 2.0 of Impl] {$\exists y$};
\node (and1) [below of=E_x] {$\wedge$};
\node (p1) [below left=1.0 and 1.2 of and1] {$P(g(x), y)$};
\node (p2) [below right=1.0 and 1.2 of and1] {$P(z, x)$};
\node (and2) [below=of E_y] {$\wedge$};
\node (E_z) [below left=1.0 and 1.2 of and2] {$\exists z$};
\node (z) [below=of E_z] {$z$};
\node (p3) [below right=1.0 and 1.5 of and2] {$P(g(y), f(x,z))$};
\end{tikzpicture}
\subsection{} %3.
\subsubsection{} %a
Für mindestens ($\exists x$) einen Tag (x) im Sommer gilt, dass die Sonne scheint ($P_{1}(x)$) und mindestens ($\exists y$) ein Rockkonzert ($Q_{1}(y)$) aufgeführt wird ($R_{1}(x,y)$).
\subsubsection{} %b
Für jeden Tag ($\forall x$) im Sommer gilt, dass wenn die Temperaturen über 35$^{\circ}$ steigen, ($P_{2}(x)$) es für alle ($\forall y$) Schüler, die unter 16 Jahren sind ($Q_{2}(y)$), hitzefrei ($R_{2}(x,y)$) gibt.
\subsubsection{} %c
An Tag ($\forall x$), der frei ist ($P_{3}(x)$), gehen alle ($\forall y$) Schüler, die zuhause sind ($R_{3}(x,y)$), zum Kino ($Q_{3}(y)$).
\subsubsection{} %d
\subsubsection{} %e
\section{} %11.4
\def\thesubsubsection{\arabic{subsubsection}.}
\setcounter{subsubsection}{0}
\subsubsection{} %1
$F_{41} = (P(x) \Rightarrow \exists x (R(x,x) \wedge \neg P(x)))$\\
\begin{tabular}{c|c|c|c|c|c|c|c|c}
& P & R & x & P(x) & R(x,x) & $\neg$P(x) & $\exists x (R(x,x) \wedge \neg P(x)) $ & $F_{41}$ \\
\hline
$A_{4}$ & $\{3,9\}$ & $\{(3,9),(6,6), (6,9), (9,9)\}$ & 9 & 1 & & & & 0 \\
\hline
$A_{4[x/3]}$ & $\{3,9\}$ & $\{(3,9),(6,6), (6,9), (9,9)\}$ & 3 & 1 & 0 & 0 & 0 \\
$A_{4[x/6]}$ & $\{3,9\}$ & $\{(3,9),(6,6), (6,9), (9,9)\}$ & 6 & 1 & 1 & 0 & 0 \\
$A_{4[x/9]}$ & $\{3,9\}$ & $\{(3,9),(6,6), (6,9), (9,9)\}$ & 9 & 1 & 1 & 0 & 0
\end{tabular}
\subsubsection{} %2
$F_{42} = \forall y (\exists x R(x,y) \Rightarrow \forall x R(x,y))$\\
Zur Platzersparnis wird die Spalte mit den Werten von P und R weggelassen. Die sind in allen Varianten gleich.\\
\begin{tabular}{c|c|c|c|c|c|c|c}
& x & y & R(x,y) & $\exists x R(x,y)$ & $\forall x R(x,y)$ & $ \exists x R(x,y) \wedge \forall x R(x,y)$ & $F_{42}$ \\
\hline
$A_{4}$ & 9 & 9 & & & & & 0 \\
\hline
$A_{4[y/3]}$ & 9 & 3 & & 0 & 0 & 0 & \\
$A_{4[y/6]}$ & 9 & 6 & & 1 & 0 & 0 & \\
$A_{4[y/9]}$ & 9 & 9 & & 1 & 1 & 1 & \\
\hline
$A_{4[y/3][x/3]}$ & 3 & 3 & 0 & & & & \\
$A_{4[y/3][x/6]}$ & 6 & 3 & 0 & & & & \\
$A_{4[y/3][x/9]}$ & 9 & 3 & 0 & & & & \\
\hline
$A_{4[y/6][x/3]}$ & 3 & 6 & 0 & & & & \\
$A_{4[y/6][x/6]}$ & 6 & 6 & 1 & & & & \\
$A_{4[y/6][x/9]}$ & 9 & 6 & 1 & & & & \\
\hline
$A_{4[y/9][x/3]}$ & 3 & 9 & 1 & & & & \\
$A_{4[y/9][x/6]}$ & 6 & 9 & 1 & & & & \\
$A_{4[y/9][x/9]}$ & 9 & 9 & 1 & & & & \\
\end{tabular}
\end{document}

146
fgi1/Aufgabenblatt2.tex Normal file
View File

@ -0,0 +1,146 @@
\documentclass[10pt,a4paper,oneside,ngerman,numbers=noenddot]{scrartcl}
\usepackage[T1]{fontenc}
\usepackage[utf8]{inputenc}
\usepackage[ngerman]{babel}
\usepackage{amsmath}
\usepackage{amsfonts}
\usepackage{amssymb}
\usepackage{paralist}
\usepackage{gauss}
\usepackage[locale=DE,exponent-product=\cdot,detect-all]{siunitx}
\usepackage{tikz}
\usetikzlibrary{automata,matrix,fadings,calc,positioning,decorations.pathreplacing,arrows,decorations.markings}
\usepackage{polynom}
\polyset{style=C, div=:,vars=x}
\pagenumbering{arabic}
\def\thesection{2.\arabic{section})}
\def\thesubsection{\arabic{subsection}.}
\def\thesubsubsection{(\roman{subsubsection})}
\setcounter{section}{3}
\makeatletter
\renewcommand*\env@matrix[1][*\c@MaxMatrixCols c]{%
\hskip -\arraycolsep
\let\@ifnextchar\new@ifnextchar
\array{#1}}
\makeatother
\begin{document}
\author{Jim Martens}
\title{Hausaufgaben zum 16. April}
\maketitle
\section{} %2.4
Der NFA ergibt sich folgendermaßen:\\
\begin{tikzpicture}[shorten >=1pt,node distance=1.1cm,on grid]
\node (00) {$0,0$};
\node (01) [below=of 00] {$0,1$};
\node (02) [below=of 01] {$0,2$};
\node (10) [left=of 00] {$1,0$};
\node (11) [left=of 01] {$1,1$};
\node (12) [below=of 11] {$1,2$};
\node (21) [left=of 11] {$2,1$};
\node (20) [left=of 21] {$2,0$};
\node (22) [left=of 12] {$2,2$};
\node (30) [left=of 20] {$3,0$};
\node (32) [left=of 22] {$3,2$};
\node (31) [left=of 32] {$3,1$};
\path[every node/.style={font=\scriptsize}]
(00) edge[->] node [below left=0.05 and 0.15 of 00] {1} (01)
(01) edge[->] node [below left=0.15 and 0.0005 of 01] {0} (11)
(01) edge[->] node [below right=0.05 and 0.15 of 01] {1} (02)
(02) edge[->,bend right=45] node [above right=0.15 and 0.1 of 02] {1} (00)
(11) edge[->] node [below left=0.1 and 0.1 of 11] {1} (12)
(02) edge[->] node [below right=0.1 and 0.1 of 02] {0} (12)
(11) edge[->] node [below left=0.15 and 0.0005 of 11] {0} (21)
(12) edge[->] node [below left=0.15 and 0.0005 of 12] {0} (22)
(21) edge[->] node [below left=0.1 and 0.1 of 21] {1} (22)
(12) edge[->,bend left=125] node [above left=1.0 and 0.1 of 12] {1} (10)
(00) edge[->] node [above left=0.15 and 0.0005 of 00] {0} (10)
(10) edge[->] node [below right=0.05 and 0.15 of 10] {1} (11)
(22) edge[->] node [below left=0.1 and 0.1 of 22] {1} (20)
(10) edge[->] node [above left=0.1 and 0.1 of 10] {0} (20)
(20) edge[->] node [below right=0.15 and 0.0005 of 20] {1} (21)
(22) edge[->] node [below left=0.15 and 0.0005 of 22] {0} (32)
(32) edge[->,bend right=45] node [below right=0.15 and 0.15 of 32] {0} (02)
(32) edge[->] node [below left=0.1 and 0.1 of 32] {1} (30)
(20) edge[->] node [below left=0.15 and 0.0005 of 20] {0} (30)
(30) edge[->,bend left=45] node [above left=0.15 and 0.1 of 30] {0} (00)
(30) edge[->] node [below left=0.05 and 0.15 of 30] {1} (31)
(31) edge[->] node [below right=0.15 and 0.005 of 31] {1} (32)
(31) edge[->,bend right=45] node [below right=0.15 and 0.0005 of 31] {0} (01);
\end{tikzpicture}
\section{} %2.5
\subsection{} %1.
\begin{alignat*}{2}
\hat{\Delta}(\{z_{s}, z_{0}\}, 01) &=& \hat{\Delta}(\underset{p \in \{z_{s}, z_{0}\}}{\bigcup} \Delta(p,0),1) \\
&=& \hat{\Delta}(\Delta(z_{s},0) \cup \Delta(z_{0},0),1) \\
&=& \hat{\Delta}(\{z_{s},z_{0}\} \cup \{z_{e}\},1) \\
&=& \hat{\Delta}(\{z_{s},z_{0},z_{e}\},1) \\
&=& \hat{\Delta}(\underset{p \in \{z_{s},z_{0},z_{e}\}}{\bigcup} \Delta(p,1), \lambda) \\
&=& \hat{\Delta}(\Delta(z_{s},1) \cup \Delta(z_{0},1) \cup \Delta(z_{e},1),\lambda) \\
&=& \hat{\Delta}(\{z_{s}\} \cup \emptyset \cup \{z_{e}\},\lambda) \\
&=& \hat{\Delta}(\{z_{s},z_{e}\},\lambda) \\
&=& \{z_{s},z_{e}\}
\end{alignat*}
\begin{alignat*}{2}
\hat{\Delta}(\{z_{0}, z_{1}\}, 01) &=& \hat{\Delta}(\underset{p \in \{z_{0}, z_{1}\}}{\bigcup} \Delta(p,0),1) \\
&=& \hat{\Delta}(\Delta(z_{0},0) \cup \Delta(z_{1},0),1) \\
&=& \hat{\Delta}(\{z_{e}\} \cup \emptyset ,1) \\
&=& \hat{\Delta}(\{z_{e}\},1) \\
&=& \hat{\Delta}(\underset{p \in \{z_{e}\}}{\bigcup} \Delta(p,1), \lambda) \\
&=& \hat{\Delta}(\Delta(z_{e},1),\lambda) \\
&=& \hat{\Delta}(\{z_{e}\},\lambda) \\
&=& \{z_{e}\}
\end{alignat*}
\subsection{} %2.
\begin{tikzpicture}[shorten >=1pt,node distance=2.8cm,on grid]
\node[initial,state] (zs) {$\{z_{s}\}$};
\node[state] (zsz0) [below=2.0 of zs] {$\{z_{s},z_{0}\}$};
\node[state] (zsz1) [below right=2.0 and 2.0 of zs] {$\{z_{s},z_{1}\}$};
\node[state,accepting] (zsz0ze) [below=2.0 of zsz0] {$\{z_{s},z_{0},z_{e}\}$};
\node[state,accepting] (zsze) [below=2.0 of zsz0ze] {$\{z_{s},z_{e}\}$};
\node[state,accepting] (zsz1ze) [below=2.0 of zsz1] {$\{z_{s},z_{1},z_{e}\}$};
\path[every node/.style={font=\scriptsize}]
(zs) edge[->] node [below left=0.05 and 0.15 of zs] {0} (zsz0)
(zs) edge[->] node [below right=0.1 and 0.2 of zs] {1} (zsz1)
(zsz1) edge[->,bend right=45] node [above right=0.1 and 0.2 of zsz1] {0} (zs)
(zsz0) edge[->,bend left=45] node [above left=0.1 and 0.2 of zsz0] {1} (zs)
(zsz0) edge[->] node [below left=0.05 and 0.1 of zsz0] {0} (zsz0ze)
(zsz1) edge[->] node [below right=0.05 and 0.1 of zsz1] {1} (zsz1ze)
(zsz0ze) edge[->,loop left] node [left=0.1 of zsz0ze] {0} (zsz0ze)
(zsz1ze) edge[->,loop right] node [right=0.1 of zsz1ze] {1} (zsz1ze)
(zsz0ze) edge[->] node [below left=0.05 and 0.15 of zsz0ze] {1} (zsze)
(zsz1ze) edge[->] node [below right=0.1 and 0.1 of zsz1ze] {0} (zsze)
(zsze) edge[->,bend left=45] node [above left=0.1 and 0.1 of zsze] {0} (zsz0ze)
(zsze) edge[->,bend right=45] node [above right=0.1 and 0.25 of zsze] {1} (zsz1ze);
\end{tikzpicture}
\subsection{} %3.
\begin{tikzpicture}[shorten >=1pt,node distance=1.1cm,on grid]
\node (z0) {$z_{0}$};
\node (z1) [below=of z0] {$z_{1}$};
\node (z2) [right=of z0] {$z_{2}$};
\node (z3) [below=of z2] {$z_{3}$};
\path[every node/.style={font=\scriptsize}]
(z0) edge[->] node [above right=0.15 and 0.0005 of z0] {a} (z2)
(z0) edge[->,loop left] node [left=0.15 of z0] {a} (z0)
(z1) edge[->] node [below right=0.15 and 0.0005 of z1] {a} (z3)
(z1) edge[->] node [below right=0.1 and 0.1 of z1] {a} (z2)
(z2) edge[->,bend right=45] node [above left=0.2 and 0.0005 of z2] {b} (z0)
(z2) edge[->] node [below right=0.15 and 0.15 of z2] {b} (z3)
(z3) edge[->,loop right] node [right=0.15 of z3] {b} (z3);
\end{tikzpicture}\\
Es lässt sich nicht rekonstruieren, welche Zustände Start- bzw. Endzustände sind. Ebenso lässt sich nicht rekonstruieren ob es eine Kante von $z_{1}$ aus gibt, die mit einem b benutzt wird. Außerdem fehlen Infos über mögliche a-Kanten der Zustände $z_{2}$ und $z_{3}$.
\section{} %2.6
\textbf{Behauptung:} Die Aussage $L(A_{1}) = \{ab\}\{cb\}^{*} =: M_{1}$ gilt für den Automaten $A_{1}$.\\
\textbf{Beweis:} Durch vollständige Induktion. \\
Mit $A(n)$ sei die Aussage $L(A_{1}) = \{ab(cb)^{n}|n \in \mathbb{N} \} =: M_{1}$ bezeichnet. \\
\underline{Induktionsanfang:} $A(0)$ ist wahr, da $ab$ ein vom Automaten $A_{1}$ akzeptiertes Wort ist und sich der Automat nach dem Lesen des Wortes im Endzustand befindet.\\\\
\underline{Induktionsannahme:} Für ein beliebig fest gewähltes $n \in \mathbb{N}$ gilt $A(n)$, d. h. es gelte $L(A_{1}) = \{ab(cb)^{n}|n \in \mathbb{N} \} =: M_{1}$.\\\\
\underline{Zu zeigen:} $A(n+1)$ gilt, d. h. $L(A_{1}) = \{ab(cb)^{n+1}|n \in \mathbb{N} \} =: M_{1}$ gilt.\\\\
\underline{Induktionsschluss:}\\
Sei $L(A_{1}) = \{ab(cb)^{n+1}|n \in \mathbb{N} \} =: M_{1}$. Dann gilt analog $L(A_{1}) = \{ab(cb)^{n}(cb)|n \in \mathbb{N} \} =: M_{1}$ bzw. $L(A_{1}) = \{ab(cb)^{n}|n \in \mathbb{N} \}\{cb\} =: M_{1}$. \\
Dabei gilt $L(A_{1}) = \{ab(cb)^{n}|n \in \mathbb{N} \}$ aufgrund der Induktionsannahme.\\
Demnach befindet sich der Automat nach Lesen des Wortes $ab(cb)^{n}, n \in \mathbb{N}$ im Endzustand. Liest man nun ein weiteres $cb$, so befindet sich der Automat wieder im Endzustand. Somit gilt $L(A_{1}) = \{ab(cb)^{n+1}|n \in \mathbb{N} \} =: M_{1}$.\\\\
Aus dem Induktionsanfang und dem Induktionsschritt ergibt sich nach dem Induktionsprinzip die Behauptung. \hfill $\Box$.
\end{document}

179
fgi1/Aufgabenblatt3.tex Normal file
View File

@ -0,0 +1,179 @@
\documentclass[10pt,a4paper,oneside,ngerman,numbers=noenddot]{scrartcl}
\usepackage[T1]{fontenc}
\usepackage[utf8]{inputenc}
\usepackage[ngerman]{babel}
\usepackage{amsmath}
\usepackage{amsfonts}
\usepackage{amssymb}
\usepackage{paralist}
\usepackage{gauss}
\usepackage{stmaryrd}
\usepackage[locale=DE,exponent-product=\cdot,detect-all]{siunitx}
\usepackage{tikz}
\usetikzlibrary{automata,matrix,fadings,calc,positioning,decorations.pathreplacing,arrows,decorations.markings}
\usepackage{polynom}
\polyset{style=C, div=:,vars=x}
\pagenumbering{arabic}
\def\thesection{3.\arabic{section})}
\def\thesubsection{\arabic{subsection}.}
\def\thesubsubsection{(\roman{subsubsection})}
\setcounter{section}{3}
\makeatletter
\renewcommand*\env@matrix[1][*\c@MaxMatrixCols c]{%
\hskip -\arraycolsep
\let\@ifnextchar\new@ifnextchar
\array{#1}}
\makeatother
\begin{document}
\author{Jim Martens}
\title{Hausaufgaben zum 23. April}
\maketitle
\section{} %3.4
\subsection{} %1.
\begin{alignat*}{2}
L_{1} &=& \{a^{2n}bcba^{2n} | n \geq 0 \}
\end{alignat*}
Angenommen $L_{1} \in REG$. Sei $n$ die PL-Zahl. Betrachte $z = a^{2n}bcba^{2n} \in L_{1}$. Also $|z| = 4n + 3 \geq n$. Also existiert eine Zerlegung $z = uvw$ mit (i), (ii) und (iii).\\
\begin{enumerate}[i)]
\item $|uv| \leq n$
\item $|v| \geq 1$
\item $\forall i \in \mathbb{N}: uv^{i}w \in L$
\end{enumerate}
\begin{alignat*}{2}
\overset{i)}{\Rightarrow} uv \in \{a\}^{*} &\Rightarrow & u,v \in \{a\}^{*} \\
&\overset{ii)}{\Rightarrow} & v \in \{a\}^{+} \text{ d.h. } v=a^{m} \text{ für } 1 \leq m \leq n
\end{alignat*}\\
Mit iii) folgt für $i = 0$, dass $uv^{i}w = a^{2n-m}bcba^{2n} \not\in L_{1} \lightning$.\\
Also $L_{1} \not\in REG$.
\subsection{} %2.
\begin{alignat*}{2}
L_{2} &=& \{w \in \{0,1\}^{*} | |w|_{1} = 2 \cdot |w|_{0} \}
\end{alignat*}
Angenommen $L_{2} \in REG$. Sei $n$ die PL-Zahl. Betrachte $z = 0^{n}1^{2n} \in L_{2}$. Also $|z| = 3n \geq n$. Also existiert eine Zerlegung $z = uvw$ mit (i), (ii) und (iii).\\
\begin{enumerate}[i)]
\item $|uv| \leq n$
\item $|v| \geq 1$
\item $\forall i \in \mathbb{N}: uv^{i}w \in L$
\end{enumerate}
\begin{alignat*}{2}
\overset{i)}{\Rightarrow} uv \in \{0\}^{*} &\Rightarrow & u,v \in \{0\}^{*} \\
&\overset{ii)}{\Rightarrow} & v \in \{0\}^{+} \text{ d.h. } v=a^{m} \text{ für } 1 \leq m \leq n
\end{alignat*}\\
Mit iii) folgt für $i = 0$, dass $uv^{i}w = 0^{n-m}1^{2n} \not\in L \lightning$.\\
Also $L \not\in REG$.
\section{} %3.5
\subsection{} %1.
Die Menge $\overline{\{a\}^{*}}$ lässt sich über dem Alphabet $\Sigma$ auch so schreiben: $\Sigma^{*} \setminus \{a\}^{*}$. Die regulären Ausdrücke $A = (a|b)^{*}$ und $B = a^{*}$ beschreiben jeweils $\Sigma^{*}$ und $\{a\}^{*}$.
Der Ausdruck $C = B \cdot b^{+} \cdot A$ beschreibt die gesuchte Menge $\overline{\{a\}^{*}}$.\\
Dieser Ausdruck akzeptiert beliebig viele $a$ am Anfang (auch keine), erwartet dann mindestens ein $b$ aber auch nicht mehr und gibt einem danach die freie Wahl zwischen beliebig vielen $a$ und $b$ ohne Beschränkung. Dadurch ist gewährleistet, dass in jedem Ausdruck mindestens ein $b$ vorkommt, womit weder das leere Wort noch eine Zeichenkette, die nur aus dem Buchstaben $a$ besteht, akzeptiert werden. Somit wird kein Element von $\{a\}^{*}$ akzeptiert, welches dem Komplement eben dieser Menge entspricht.
\subsection{} %2.
Berechnung eines regulären Ausdruckes für den gegebenen DFA mithilfe des Kleene-Verfahrens.
\begin{alignat*}{2}
R_{1,1}^{0} &=& \{\epsilon , b\} \\
R_{1,2}^{0} &=& \{a\} \\
R_{1,3}^{0} &=& R_{2,1}^{0} = R_{3,1}^{0} = \emptyset \\
R_{2,2}^{0} &=& \{\epsilon \} \\
R_{2,3}^{0} &=& \{a\} \\
R_{3,2}^{0} &=& \{b\} \\
R_{3,3}^{0} &=& \{\epsilon \} \end{alignat*}
\begin{alignat*}{2}
R_{1,2}^{1} &=& R_{1,2}^{0} \cup R_{1,1}^{0} \cdot (R_{1,1}^{0})^{*} \cdot R_{1,2}^{0} \\
&=& R_{1,2}^{0} \cup (R_{1,1}^{0})^{+} \cdot R_{1,2}^{0} \\
&=& (R_{1,1}^{0})^{+} \cdot R_{1,2}^{0} \cup R_{1,2}^{0} \\
&=& (R_{1,1}^{0})^{+} \cdot R_{1,2}^{0} \\
&=& \{\epsilon , b \}^{+} \cdot \{a\} \\
&=& \{b\}^{*} \cdot \{a\} \\
&\overset{\sim}{=}& ((b)^{*} \cdot a) \\
%
R_{2,3}^{1} &=& R_{2,3}^{0} \cup R_{2,1}^{0} \cdot (R_{1,1}^{0})^{*} \cdot R_{1,3}^{0} \\
&=& R_{2,3}^{0} \cup R_{2,1}^{0} \cdot (R_{1,1}^{0})^{*} \cdot R_{1,3}^{0} \\
&=& \{a\} \cup \emptyset \cdot \{\epsilon , b\}^{*} \cdot \emptyset \\
&=& \{a\} \cup \emptyset \\
&=& \{a\} \\
&\overset{\sim}{=}& a \\
%
R_{3,2}^{1} &=& R_{3,2}^{0} \cup R_{3,1}^{0} \cdot (R_{1,1}^{0})^{*} \cdot R_{1,3}^{0} \\
&=& \{b\} \cup \emptyset \cdot \{\epsilon , b\}^{*} \cdot \emptyset \\
&=& \{b\} \cup \emptyset \cdot \{b\}^{*} \cdot \emptyset \\
&=& \{b\} \cup \emptyset \\
&=& \{b\} \\
&\overset{\sim}{=}& b \\
%
R_{2,2}^{1} &=& R_{2,2}^{0} \cup R_{2,1}^{0} \cdot (R_{1,1}^{0})^{*} \cdot R_{1,2}^{0} \\
&=& \{\epsilon \} \cup (\emptyset \cdot \{\epsilon , b\}^{*} \cdot \{a\}) \\
&=& \{\epsilon \} \cup (\emptyset \cdot \{b\}^{*} \cdot \{a\}) \\
&=& \{\epsilon \} \cup \emptyset \\
&=& \{\epsilon \} \\
&\overset{\sim}{=}& \emptyset^{*} \\
%
R_{3,3}^{1} &=& R_{3,3}^{0} \cup R_{3,1}^{0} \cdot (R_{1,1}^{0})^{*} \cdot R_{1,3}^{0} \\
&=& \{\epsilon \} \cup \emptyset \cdot \{\epsilon , b\}^{*} \cdot \emptyset \\
&=& \{\epsilon \} \cup \emptyset \cdot \{b\}^{*} \cdot \emptyset \\
&=& \{\epsilon \} \cup \emptyset \\
&=& \{\epsilon \} \\
&\overset{\sim}{=}& \emptyset^{*}
\end{alignat*}\\
\begin{alignat*}{2}
R_{1,3}^{1} &=& R_{1,3}^{0} \cup R_{1,1}^{0} \cdot (R_{1,1}^{0})^{*} \cdot R_{1,3}^{0} \\
&=& R_{1,3}^{0} \cup (R_{1,1}^{0})^{+} \cdot R_{1,3}^{0} \\
&=& (R_{1,1}^{0})^{+} \cdot R_{1,3}^{0} \cup R_{1,3}^{0}\\
&=& (R_{1,1}^{0})^{+} \cdot R_{1,3}^{0}\\
&=& \{\epsilon , b \}^{+} \cdot \emptyset \\
&=& \{b \}^{*} \cdot \emptyset \\
&=& \emptyset \\
%
R_{1,2}^{2} &=& R_{1,2}^{1} \cup R_{1,2}^{1} \cdot (R_{2,2}^{1})^{*} \cdot R_{2,2}^{1}) \\
&=& R_{1,2}^{1} \cdot (\{\epsilon \} \cup (R_{2,2}^{1})^{*} \cdot R_{2,2}^{1}) \\
&=& R_{1,2}^{1} \cdot (\{\epsilon \} \cup (R_{2,2}^{1})^{+}) \\
&=& R_{1,2}^{1} \cdot (R_{2,2}^{1})^{*} \\
&=& (\{b\}^{*} \cdot \{a\}) \cdot \{\epsilon \}^{*} \\
&=& \{b\}^{*} \cdot \{a\}\\
&\overset{\sim}{=}& ((b)^{*} \cdot a) \\
%
R_{1,3}^{2} &=& R_{1,3}^{1} \cup R_{1,2}^{1} \cdot (R_{2,2}^{1})^{*} \cdot R_{2,3}^{1} \\
&=& \emptyset \cup (\{a\} \cdot \{\epsilon \}^{*} \cdot \{a\}) \\
&=& \emptyset \cup (\{a\} \cdot \{a\}) \\
&=& \emptyset \cup \{aa\} \\
&=& \{aa\} \\
&\overset{\sim}{=}& aa \\
%
R_{3,3}^{2} &=& R_{3,3}^{1} \cup R_{3,2}^{1} \cdot (R_{2,2}^{1})^{*} \cdot R_{2,3}^{1} \\
&=& \{\epsilon \} \cup (\{b\} \cdot \{\epsilon \}^{*} \cdot \{a\}) \\
&=& \{\epsilon \} \cup (\{b\} \cdot \{a\}) \\
&=& \{\epsilon \} \cup \{ba\} \\
&=& \{\epsilon , ba\} \\
&\overset{\sim}{=}& \emptyset^{*} + b \cdot a
\end{alignat*}\\
\begin{alignat*}{2}
R_{3,2}^{2} &=& R_{3,2}^{1} \cup R_{3,2}^{1} \cdot (R_{2,2}^{1})^{*} \cdot R_{2,2}^{1} \\
&=& R_{3,2}^{1} \cdot (\{\epsilon \} \cup (R_{2,2}^{1})^{*} \cdot R_{2,2}^{1}) \\
&=& R_{3,2}^{1} \cdot (\{\epsilon \} \cup (R_{2,2}^{1})^{+}) \\
&=& R_{3,2}^{1} \cdot (R_{2,2}^{1})^{*} \\
&=& \{b\} \cdot \{\epsilon \}^{*} \\
&=& \{b\} \\
&\overset{\sim}{=}& b \\
%
R_{1,3}^{3} &=& R_{1,3}^{2} \cup R_{1,3}^{2} \cdot (R_{3,3}^{2})^{*} \cdot R_{3,3}^{2} \\
&=& R_{1,3}^{2} \cdot (\{\epsilon \} \cup (R_{3,3}^{2})^{*} \cdot R_{3,3}^{2}) \\
&=& R_{1,3}^{2} \cdot (\{\epsilon \} \cup (R_{3,3}^{2})^{+}) \\
&=& R_{1,3}^{2} \cdot (R_{3,3}^{2})^{*} \\
&=& \{aa\} \cdot \{\epsilon , ba\}^{*} \\
&=& \{aa\} \cdot \{ba\}^{*} \\
&\overset{\sim}{=}& a \cdot a \cdot (b \cdot a)^{*} \\
%
R_{1,2}^{3} &=& R_{1,2}^{2} \cup R_{1,3}^{2} \cdot (R_{3,3}^{2})^{*} \cdot R_{3,2}^{2} \\
&=& (\{b\}^{*} \cdot \{a\}) \cup (\{aa\} \cdot \{\epsilon , ba\}^{*} \cdot \{b\}) \\
&=& (\{b\}^{*} \cdot \{a\}) \cup (\{aa\} \cdot \{ba\}^{*} \cdot \{b\}) \\
&\overset{\sim}{=}& (b^{*} \cdot a) + (aa \cdot (b \cdot a)^{*} \cdot b) \\
\end{alignat*}
Der reguläre Ausdruck ergibt sich aus der Vereinigung von $R_{1,3}^{3}$ und $R_{1,2}^{3}$. Es ergibt sich daher folgendes:\\
\begin{alignat*}{2}
R_{1,3}^{3} \cup R_{1,2}^{3} &=& (\{aa\} \cdot \{ba\}^{*}) \cup ((\{b\}^{*} \cdot \{a\}) \cup (\{aa\} \cdot \{ba\}^{*} \cdot \{b\})) \\
&\overset{\sim}{=}& (aa \cdot (ba)^{*}) + (b^{*} \cdot a) + (aa \cdot (ba)^{*} \cdot b)
\end{alignat*}
Der reguläre Ausdruck ist $(aa \cdot (ba)^{*}) + (b^{*} \cdot a) + (aa \cdot (ba)^{*} \cdot b)$.
\section{} %3.6
\subsection{} %1.
\subsection{} %2.
\end{document}

170
fgi1/Aufgabenblatt4.tex Normal file
View File

@ -0,0 +1,170 @@
\documentclass[10pt,a4paper,oneside,ngerman,numbers=noenddot]{scrartcl}
\usepackage[T1]{fontenc}
\usepackage[utf8]{inputenc}
\usepackage[ngerman]{babel}
\usepackage{amsmath}
\usepackage{amsfonts}
\usepackage{amssymb}
\usepackage{paralist}
\usepackage{gauss}
\usepackage{stmaryrd}
\usepackage[locale=DE,exponent-product=\cdot,detect-all]{siunitx}
\usepackage{tikz}
\usetikzlibrary{automata,matrix,fadings,calc,positioning,decorations.pathreplacing,arrows,decorations.markings}
\usepackage{polynom}
\polyset{style=C, div=:,vars=x}
\pagenumbering{arabic}
\def\thesection{4.\arabic{section})}
\def\thesubsection{\arabic{subsection}.}
\def\thesubsubsection{(\roman{subsubsection})}
\setcounter{section}{3}
\makeatletter
\renewcommand*\env@matrix[1][*\c@MaxMatrixCols c]{%
\hskip -\arraycolsep
\let\@ifnextchar\new@ifnextchar
\array{#1}}
\makeatother
\begin{document}
\author{Jim Martens}
\title{Hausaufgaben zum 30. April}
\maketitle
\section{} %4.4
\subsection{} %1.
$G_{1}$:\\
\begin{alignat*}{2}
S &\rightarrow & aS | AB \\
A &\rightarrow & aAb | \lambda \\
B &\rightarrow & bBc | \lambda
\end{alignat*}
Ich behaupte $L(G_{1}) = \{a^{i + j}b^{i + k}c^{k}|i,j,k \geq 0\} =: M$.
Zunächst zeige ich $L(G_{1}) \subseteq M$. Sei dazu $w \in L(G_{1})$, d.h. es gilt $S \overset{*}{\Longrightarrow} w$. Wie kann $w \in V_{T}^{*}$ entstanden sein? Durch $j$-malige Anwendung der Produktion $S \rightarrow aS$ erhält man zunächst $S \overset{*}{\Longrightarrow} a^{j}S$. Anschließend muss die Produktion $S \rightarrow AB$ benutzt werden. Dies führt zu $S \overset{*}{\Longrightarrow} a^{j}S \overset{*}{\Longrightarrow} a^{j}AB$. Durch $i$-malige Anwendung der Produktion $A \rightarrow aAb$ erhält man $S \overset{*}{\Longrightarrow} a^{j}S \overset{*}{\Longrightarrow} a^{j}AB \overset{*}{\Longrightarrow} a^{j}a^{i}Ab^{i}B$. Dann muss die Produktion $A \rightarrow \lambda$ gewählt werden. Es ergibt sich daher
$S \overset{*}{\Longrightarrow} a^{j}S \overset{*}{\Longrightarrow} a^{j}AB \overset{*}{\Longrightarrow} a^{j}a^{i}Ab^{i}B \Longrightarrow a^{j}a^{i}b^{i}B$. Durch $k$-malige Anwendung der Produktion $B \rightarrow bBc$ erhält man $S \overset{*}{\Longrightarrow} a^{j}S \overset{*}{\Longrightarrow} a^{j}AB \overset{*}{\Longrightarrow} a^{j}a^{i}Ab^{i}B \Longrightarrow a^{j}a^{i}b^{i}B \overset{*}{\Longrightarrow} a^{j}a^{i}b^{i}b^{k}Bc^{k}$. Abschließend muss die Produktion $B \rightarrow \lambda$ gewählt werden.\\
\\
Insgesamt muss $w$ damit die Form $a^{j}a^{i}b^{i}b^{k}c^{k} = a^{i + j}b^{i + k}c^{k}$ haben. Es gilt also $w \in M$.\\
\\
Sei umgekehrt $w \in M$. Hat $w$ die Form $a^{i + j}b^{i + k}c^{k} = a^{j}a^{i}b^{i}b^{k}c^{k}$ für $i,j,k \geq 0$, so kann $w$ durch $j$-malige Anwendung der Regel $S \rightarrow aS$, anschließende Anwendung der Regel $S \rightarrow AB$, $i$-malige Anwendung der Regel $A \rightarrow aAb$ mit anschließender Anwendung von $A \rightarrow \lambda$, $k$-maliger Anwendung von $B \rightarrow bBc$ und anschließender Anwendung von $B \rightarrow \lambda$ abgeleitet werden. Damit gilt für jedes $w \in M$ also auch $w \in L(G_{1})$ und die Behauptung ist gezeigt.
\subsection{} %2.
$G_{2}$:\\
\begin{alignat*}{2}
S &\rightarrow & 0ABD | B1CD | A1CE | AE2D \\
A &\rightarrow & 0A | \lambda \\
B &\rightarrow & 0B1 | \lambda \\
C &\rightarrow & 1C | \lambda \\
D &\rightarrow & 2D | \lambda \\
E &\rightarrow & 1E2 | \lambda
\end{alignat*}
Ich behaupte $L(G_{2}) = \{0^{i + j}1^{i}2^{n}, 0^{i}1^{i + k}2^{n}, 0^{n}1^{i + l}2^{i}, 0^{n}1^{i}2^{i + m}|i,n \geq 0 \wedge j,k,l,m \geq 1\} =: M$.\\
Zunächst zeige ich $L(G_{2}) \subseteq M$. Sei dazu $w \in L(G_{2})$, d.h. es gilt $S \overset{*}{\Longrightarrow} w$. Wie kann $w \in V_{T}^{*}$ entstanden sein? Es gibt vier Fälle für Wörter von $L_{2}$:
\begin{enumerate}
\item $|w|_{0} > |w|_{1}$ \\
\item $|w|_{0} < |w|_{1}$ \\
\item $|w|_{1} > |w|_{2}$ \\
\item $|w|_{1} < |w|_{2}$
\end{enumerate}
Es ist natürlich möglich, dass mehrere von diesen Fällen zur gleichen Zeit zutreffen, allerdings muss immer mindestens einer zutreffen. Die jeweils nicht genannten Anzahlen sind beliebig.
Für den ersten Fall ergibt sich folgende Ableitung:\\
Zunächst wird die Produktion $S \rightarrow 0ABD$ gewählt. Durch $j-1$-malige Anwendung der Produktion $A \rightarrow 0A$ erhält man zunächst $S \overset{*}{\Longrightarrow} 00^{j-1}ABD$. Als nächstes wird die Produktion $A \rightarrow \lambda$ gewählt. Durch $i$-malige Anwendung der Produktion $B \rightarrow 0B1$ erhält man $S \overset{*}{\Longrightarrow} 00^{j-1}ABD \overset{*}{\Longrightarrow} 00^{j-1}0^{i}B1^{i}D$. Es wird die Produktion $B \rightarrow \lambda$ gewählt. Durch $n$-malige Anwendung von $D \rightarrow 2D$ erhält man $S \overset{*}{\Longrightarrow} 00^{j-1}ABD \overset{*}{\Longrightarrow} 00^{j-1}0^{i}B1^{i}D \overset{*}{\Longrightarrow} 00^{j-1}0^{i}1^{i}2^{n}D$. Abschließend wird $D \rightarrow \lambda$ gewählt. Das Wort $w$ hat demnach die Form $00^{j-1}0^{i}1^{i}2^{n} = 0^{i + j}1^{i}2^{n}$ mit $i, n \geq 0 \wedge j \geq 1$.\\
\\
Für den zweiten Fall ergibt sich folgende Ableitung:\\
Zunächst wird die Produktion $S \rightarrow B1CD$ gewählt. Durch $i$-malige Anwendung der Produktion $B \rightarrow 0B1$ erhält man $S \overset{*}{\Longrightarrow}0^{i}B1^{i}1CD$. Es wird die Produktion $B \rightarrow \lambda$ gewählt. Durch $k-1$-malige Anwendung von $C \rightarrow 1C$ erhält man $S \overset{*}{\Longrightarrow}0^{i}B1^{i}1CD \overset{*}{\Longrightarrow} 0^{i}1^{i}11^{k-1}CD$. Durch $n$-malige Anwendung von $D \rightarrow 2D$ erhält man $S \overset{*}{\Longrightarrow}0^{i}B1^{i}1CD \overset{*}{\Longrightarrow} 0^{i}1^{i}11^{k-1}CD \overset{*}{\Longrightarrow} 0^{i}1^{i}11^{k-1}2^{n}D$. Abschließend wird $D \rightarrow \lambda$ gewählt. Das Wort $w$ hat demnach die Form $0^{i}1^{i}11^{k-1}2^{n} = 0^{i}1^{i + k}2^{n}$ mit $i, n \geq 0 \wedge k \geq 1$.\\
\\
Für den dritten Fall ergibt sich diese Ableitung:\\
Zunächst wird die Produktion $S \rightarrow A1CE$ gewählt. Durch $n$-malige Anwendung der Produktion $A \rightarrow 0A$ erhält man zunächst $S \overset{*}{\Longrightarrow} 0^{n}A1CE$. Direkt danach folgt die Produktion $A \rightarrow \lambda$. Durch $l-1$-malige Anwendung der Produktion $C \rightarrow 1C$ erhält man $S \overset{*}{\Longrightarrow} 0^{n}A1CE \overset{*}{\Longrightarrow} 0^{n}11^{l-1}CE$. Anschließend folgt die Produktion $C \rightarrow \lambda$. Mit $i$-maliger Anwendung der Produktion $E \rightarrow 1E2$ erhält man $S \overset{*}{\Longrightarrow} 0^{n}A1CE \overset{*}{\Longrightarrow} 0^{n}11^{l-1}CE \overset{*}{\Longrightarrow} 0^{n}11^{l-1}1^{i}E2^{i}$. Abschließend wird die Produktion $E \rightarrow \lambda$ angewendet. Das Wort $w$ hat demnach die Form $0^{n}11^{l-1}1^{i}2^{i} = 0^{n}1^{i + l}2^{i}$ mit $i,n \geq 0 \wedge l \geq 1$. \\
\\
Für den vierten Fall ergibt sich diese Ableitung:\\
Zunächst wird die Produktion $S \rightarrow AE2D$ gewählt. Durch $n$-malige Anwendung der Produktion $A \rightarrow 0A$ erhält man zunächst $S \overset{*}{\Longrightarrow} 0^{n}AE2D$. Direkt danach folgt die Produktion $A \rightarrow \lambda$. Mit $i$-maliger Anwendung der Produktion $E \rightarrow 1E2$ erhält man $S \overset{*}{\Longrightarrow} 0^{n}AE2D \overset{*}{\Longrightarrow} 0^{n}1^{i}E2^{i}2D$. Abschließend wird die Produktion $E \rightarrow \lambda$ angewendet. Durch $m-1$-malige Anwendung der Produktion $D \rightarrow 2D$ erhält man $S \overset{*}{\Longrightarrow} 0^{n}AE2D \overset{*}{\Longrightarrow} 0^{n}1^{i}E2^{i}2D \overset{*}{\Longrightarrow} 0^{n}1^{i}2^{i}22^{m-1}D$. Abschließend wird die Produktion $D \rightarrow \lambda$ gewählt. Das Wort $w$ hat demnach die Form $0^{n}1^{i}2^{i}22^{m-1} = 0^{n}1^{i}2^{i + m}$ mit $i,n \geq 0 \wedge m \geq 1$. \\
\\
Insgesamt muss $w$ damit die Form $0^{i + j}1^{i}2^{n}, 0^{i}1^{i + k}2^{n}, 0^{n}1^{i + l}2^{i}$ oder $0^{n}1^{i}2^{i + m}$ haben. Es gilt also $w \in M$.\\
\\
Sei umgekehrt $w \in M$. Hat $w$ die Form $0^{i + j}1^{i}2^{n}$, so kann $w$ durch die im ersten Fall beschriebene Ableitungsfolge abgeleitet werden. Hat $w$ die Form $0^{i}1^{i + k}2^{n}$, so kann $w$ durch die im zweiten Fall beschriebene Ableitungsfolge abgeleitet werden. Hat $w$ die Form $0^{n}1^{i + l}2^{i}$, so kann $w$ durch die im dritten Fall beschriebene Ableitungsfolge abgeleitet werden. Hat $w$ die Form $0^{n}1^{i}2^{i + m}$, so kann $w$ durch die im vierten Fall beschriebene Ableitungsfolge abgeleitet werden. Damit gilt für jedes $w \in M$ also auch $w \in L(G_{2})$ und die Behauptung ist gezeigt.
\section{} %4.5
Zunächst werden alle $\lambda$-Produktionen nach dem Verfahren von Theorem 9.12 entfernt.
\begin{alignat*}{2}
M_{0} &=& \{C\} \\
M_{1} &=& \{C,F,S\} \\
M_{2} &=& \{C,F,S\} = M_{1} \\
V_{\lambda} &=& M_{2}
\end{alignat*}
Daraus ergibt sich folgende Grammatik:\\
\begin{alignat*}{2}
S &\rightarrow & AB | AB1B | AD | DE | C \\
A &\rightarrow & EBEE \\
B &\rightarrow & A | E | SE \\
D &\rightarrow & CBG | BG | 0G | 1D \\
E &\rightarrow & 0 | 1 \\
F &\rightarrow & 0E | EF | CC | C | E
\end{alignat*}\\
Anschließend wird nach dem Verfahren von Theorem 9.11 reduziert. Zunächst werden dabei alle unproduktiven Nonterminale entfernt.\\
\begin{alignat*}{2}
M_{0} &=& \{0,1\} \\
M_{1} &=& \{0,1,E\} \\
M_{2} &=& \{0,1,E,F,B\} \\
M_{3} &=& \{0,1,E,F,B,A\} \\
M_{4} &=& \{0,1,E,F,B,A,S\} \\
M_{5} &=& \{0,1,E,F,B,A,S\} = M_{4}
\end{alignat*}
Es ergibt sich folgende Grammatik:\\
\begin{alignat*}{2}
S &\rightarrow & AB | AB1B \\
A &\rightarrow & EBEE \\
B &\rightarrow & A | E | SE \\
E &\rightarrow & 0 | 1 \\
F &\rightarrow & 0E | EF | E
\end{alignat*}
Daraufhin werden alle Produktionen entfernt, die auf der linken oder rechten Seite unerreichbare Nonterminale haben.
\begin{alignat*}{2}
M_{0} &=& \{S\} \\
M_{1} &=& \{S,A,B\} \\
M_{2} &=& \{S,A,B,E\} \\
M_{3} &=& \{S,A,B,E\} = M_{2}
\end{alignat*}
Es ergibt sich die Grammatik:\\
\begin{alignat*}{2}
S &\rightarrow & AB | AB1B \\
A &\rightarrow & EBEE \\
B &\rightarrow & A | E | SE \\
E &\rightarrow & 0 | 1
\end{alignat*}
Im dritten Schritt werden die Kettenregeln entfernt. Daraus ergibt sich diese Grammatik:\\
\begin{alignat*}{2}
S &\rightarrow & AB | AB1B \\
A &\rightarrow & EBEE \\
B &\rightarrow & EBEE | 0 | 1 | SE \\
E &\rightarrow & 0 | 1
\end{alignat*}
Im vierten Schritt werden lange Terminalregeln ersetzt. Daraus ergibt sich diese Grammatik:\\
\begin{alignat*}{2}
S &\rightarrow & AB | AB\left\langle 1 \right\rangle B \\
A &\rightarrow & EBEE \\
B &\rightarrow & EBEE | 0 | 1 | SE \\
E &\rightarrow & 0 | 1 \\
\left\langle 1 \right\rangle &\rightarrow & 1
\end{alignat*}
Im fünften Schritt werden zu lange Regeln verkürzt. Daraus ergibt sich diese Grammatik:\\
\begin{alignat*}{2}
S &\rightarrow & AB | \left\langle AB \right\rangle \left\langle \left\langle 1 \right\rangle B \right\rangle \\
\left\langle AB \right\rangle &\rightarrow & AB \\
\left\langle \left\langle 1 \right\rangle B \right\rangle &\rightarrow &\left\langle 1 \right\rangle B \\
A &\rightarrow & \left\langle EB \right\rangle \left\langle EE \right\rangle \\
\left\langle EB \right\rangle &\rightarrow & EB \\
\left\langle EE \right\rangle &\rightarrow & EE \\
B &\rightarrow & \left\langle EB \right\rangle \left\langle EE \right\rangle | 0 | 1 | SE \\
E &\rightarrow & 0 | 1 \\
\left\langle 1 \right\rangle &\rightarrow & 1
\end{alignat*}
Im sechsten Schritt wird die ursprüngliche Sprache wiederhergestellt, indem eine $\lambda$-Regel hinzugenommen wird. Daraus ergibt sich diese Grammatik:\\
\begin{alignat*}{2}
S_{neu} &\rightarrow & \lambda | AB | \left\langle AB \right\rangle \left\langle \left\langle 1 \right\rangle B \right\rangle \\
\left\langle AB \right\rangle &\rightarrow & AB \\
\left\langle \left\langle 1 \right\rangle B \right\rangle &\rightarrow &\left\langle 1 \right\rangle B \\
A &\rightarrow & \left\langle EB \right\rangle \left\langle EE \right\rangle \\
\left\langle EB \right\rangle &\rightarrow & EB \\
\left\langle EE \right\rangle &\rightarrow & EE \\
B &\rightarrow & \left\langle EB \right\rangle \left\langle EE \right\rangle | 0 | 1 | SE \\
E &\rightarrow & 0 | 1 \\
\left\langle 1 \right\rangle &\rightarrow & 1
\end{alignat*}
\section{} %4.6
\end{document}

93
fgi1/Aufgabenblatt5.tex Normal file
View File

@ -0,0 +1,93 @@
\documentclass[10pt,a4paper,oneside,ngerman,numbers=noenddot]{scrartcl}
\usepackage[T1]{fontenc}
\usepackage[utf8]{inputenc}
\usepackage[ngerman]{babel}
\usepackage{amsmath}
\usepackage{amsfonts}
\usepackage{amssymb}
\usepackage{paralist}
\usepackage{gauss}
\usepackage{stmaryrd}
\usepackage[locale=DE,exponent-product=\cdot,detect-all]{siunitx}
\usepackage{tikz}
\usetikzlibrary{automata,matrix,fadings,calc,positioning,decorations.pathreplacing,arrows,decorations.markings}
\usepackage{polynom}
\polyset{style=C, div=:,vars=x}
\pagenumbering{arabic}
\def\thesection{5.\arabic{section})}
\def\thesubsection{\arabic{subsection}.}
\def\thesubsubsection{(\roman{subsubsection})}
\setcounter{section}{3}
\makeatletter
\renewcommand*\env@matrix[1][*\c@MaxMatrixCols c]{%
\hskip -\arraycolsep
\let\@ifnextchar\new@ifnextchar
\array{#1}}
\makeatother
\begin{document}
\author{Jim Martens}
\title{Hausaufgaben zum 7. Mai}
\maketitle
\section{} %5.4
\subsection{} %1.
$G_{1}$:\\
\begin{alignat*}{2}
S &\rightarrow & aSb | A \\
A &\rightarrow & bAa | \lambda
\end{alignat*}
Behauptung: Es gilt $L(G_{1}) = L_{1}$.\\
Zunächst zeige ich $L(G_{1}) \subseteq L_{1}$. Sei dazu $w \in L(G_{1})$, d.h. es gilt $S \overset{*}{\Longrightarrow} w$. Durch $n$-malige Anwendung der Produktion $S \rightarrow aSb$ erhält man zunächst $S \overset{*}{\Longrightarrow} a^{n}Sb^{n}$. Anschließend muss die Produktion $S \rightarrow A$ gewählt werden. Durch $m$-malige Anwendung der Regel $A \rightarrow bAa$ erhält man $S \overset{*}{\Longrightarrow} a^{n}Sb^{n} \overset{*}{\Longrightarrow} a^{n}b^{m}Aa^{m}b^{n}$. Abschließend wird die Produktion $A \rightarrow \lambda$ gewählt. Insgesamt muss $w$ damit die Form $a^{n}b^{m}a^{m}b^{n}$ haben. Es gilt also $w \in L_{1}$.\\
\\
Sei umgekehrt $w \in L_{1}$. Hat $w$ die Form $a^{n}b^{m}a^{m}b^{n}$ für $n,m \geq 0$, so kann $w$ durch $n$-malige Anwendung der Produktion $S \rightarrow aSb$, Anwendung der Regel $S \rightarrow A$, $m$-malige Anwendung von $A \rightarrow bAa$ und abschließende Anwendung von $A \rightarrow \lambda$ abgeleitet werden.\\
Damit gilt für jedes $w \in L_{1}$ demnach auch $w \in L(G_{1})$ und die Behauptung ist gezeigt.\\
\\
Bei Anwendung des Pumping-Lemmas kommt es bei der Wahl des Wortes zu Problemen. Bei einer kontextfreien Sprache gilt das Pumping-Lemma für alle Wörter der Sprache. Möchte man dies nun zeigen, so müsste man unendlich viele Wörter zeigen, was nicht machbar ist.\\
Sollte man einen Widerspruch zeigen wollen, so wird man kein Wort der Sprache finden, bei dem es einen gibt, denn bei einer kontextfreien Sprache gilt das Pumping-Lemma für jedes Wort.
\subsection{} %2.
\begin{alignat*}{2}
L_{2} &=& \{a^{n}b^{m}a^{n}b^{m} | n,m \geq 0\}
\end{alignat*}\\
Angenommen $L_{2}$ wäre kontextfrei.\\
Sei $n$ die Zahl aus dem Pumping Lemma. Wähle $z = a^{n}b^{n}a^{n}b^{n}$ mit $n \in \mathbb{N}$.\\
Die ersten $a$s seien durch $a_{1}$ markiert und die ersten $b$s durch $b_{1}$. Die jeweils zweiten Blöcke durch den Index $2$.\\
Also $z \in L_{2}, |z| = 4n$.\\
Also existiert eine Zerlegung $z = uvwxy$ mit $(i)$ und $(ii)$. Ich führe nun jede dieser Zerlegungen zu einem Widerspruch mit der dritten Eigenschaft des Pumping Lemmas.\\
\begin{enumerate}[i)]
\item $|vx| \geq 1$
\item $|vwx| \leq n$
\item $\forall i \geq 0:uv^{i}wx^{i}y \in L$
\end{enumerate}
Zunächst gibt es wegen $|vwx| \leq n$ und der Form von $z$ sieben Fälle zu unterscheiden: $vwx \in \{a_{1}\}^{*}, vwx \in \{a_{1}\}^{*}\{b_{1}\}^{*}, vwx \in \{b_{1}\}^{*}, vwx \in \{b_{1}\}^{*}\{a_{2}\}^{*}, vwx \in \{a_{2}\}^{*}, vwx \in \{a_{2}\}^{*}\{b_{2}\}^{*} \text{ und } vwx \in \{b_{2}\}^{*}$. In den Fällen $vwx \in \{a_{1}\}^{*}, vwx \in \{b_{1}\}^{*}, vwx \in \{a_{2}\}^{*}$ und $vwx \in \{b_{2}\}^{*}$ führt wegen $|vx| \geq 1$ die Betrachtung von $uv^{2}wx^{2}y$ bereits zum Widerspruch. Es ist dann $uv^{2}wx^{2}y = a^{n+j}b^{n}a^{n}b^{n}$, $uv^{2}wx^{2}y = a^{n}b^{n+j}a^{n}b^{n}$, $uv^{2}wx^{2}y = a^{n}b^{n}a^{n+j}b^{n}$ bzw. $uv^{2}wx^{2}y = a^{n}b^{n}a^{n}b^{n+j}$ mit $j > 0$ und folglich gilt nicht mehr, dass der Exponent von den beiden $a$- bzw. $b$-Blöcken gleich sein muss.\\
\\
Gilt in $vwx \in \{a_{1}\}^{*}\{b_{1}\}^{*}, vwx \in \{b_{1}\}^{*}\{a_{2}\}^{*}$ bzw. $vwx \in \{a_{2}\}^{*}\{b_{2}\}^{*}$, dass $v$ oder $x$ bereits aus mehr als einem Symbol aufgebaut sind (also selbst in $\{a\}^{+}\{b\}^{+}$ bzw. $\{b\}^{+}\{a\}^{+}$ sind), so führt $uv^{2}wx^{2}y$ sofort zu einem Widerspruch, da dieses Wort dann nicht einmal mehr in $\{a\}^{*}\{b\}^{*}\{a\}^{*}\{b\}^{*}$ ist (und damit ganz bestimmt nicht in $L_{2}$).\\
\\
Ist jedoch im Fall $vwx \in \{a_{1}\}^{*}\{b_{1}\}^{*}$ das $v$ aus den $a$s und das $x$ in den $b$s, so führt ähnlich wie bei den zuerst diskutierten Fällen die Betrachtung von $uv^{2}wx^{2}y = a^{n+j}b^{n+k}a^{n}b^{n}$ zu einem Widerspruch ($n+j = n$ bzw. $n+k = n$ gilt nicht, da wegen $|vx| \geq 1$ auch $j+k \geq 1$ sein muss). Der Fall $vwx \in \{a_{2}\}^{*}\{b_{2}\}^{*}$ führt analog zu einem Widerspruch (lediglich mit den beiden hinteren Blöcken).\\
Ist im Fall von $vwx \in \{b_{1}\}^{*}\{a_{2}\}^{*}$ das $v$ in den $b$s und das $x$ in den $a$s, so führt ebenfalls ähnlich zu den zuerst diskutierten Fällen die Betrachtung von $uv^{2}wx^{2}y = a^{n}b^{n+j}a^{n+k}b^{n}$ zu einem Widerspruch (abermals gilt $n+j = n$ bzw. $n+k = n$ nicht, da $j+k \geq 1$ sein muss).\\
\\
Damit sind alle Fälle zum Widerspruch geführt und es gibt also keine Zerlegung von $z$ in $uvwxy$ derart, dass die drei Bedingungen des Pumping Lemmas erfüllt sind. Folglich ist die ursprüngliche Annahme, $L_{2}$ wäre kontextfrei nicht haltbar und $L_{2}$ ist also nicht kontextfrei.
\section{} %5.5
\subsection{} %1.
$G_{1}$:\\
\begin{alignat*}{2}
S &\rightarrow & aSc | A \\
A &\rightarrow & bc | \lambda
\end{alignat*}
Behauptung: Es gilt $L(G_{1}) = L_{1}$.\\
Zunächst zeige ich $L(G_{1}) \subseteq L_{1}$. Sei dazu $w \in L(G_{1})$, d.h. es gilt $S \overset{*}{\Longrightarrow} w$. Durch $i$-malige Anwendung der Produktion $S \rightarrow aSc$ erhält man zunächst $S \overset{*}{\Longrightarrow} a^{i}Sc^{i}$. Anschließend muss die Produktion $S \rightarrow A$ gewählt werden. Durch $j$-malige Anwendung der Regel $A \rightarrow bAc$ erhält man $S \overset{*}{\Longrightarrow} a^{i}Sc^{i} \overset{*}{\Longrightarrow} a^{i}b^{j}Ac^{i+j}$. Abschließend wird die Produktion $A \rightarrow \lambda$ gewählt. Insgesamt muss $w$ damit die Form $a^{i}b^{j}c^{i+j} = a^{i}b^{j}c^{k}$ für $k = i+j$ haben. Es gilt also $w \in L_{1}$.\\
\\
Sei umgekehrt $w \in L_{1}$. Hat $w$ die Form $a^{i}b^{j}c^{k} = a^{i}b^{j}c^{i+j}$ für $k = i+j$, so kann $w$ durch $i$-malige Anwendung der Produktion $S \rightarrow aSc$, Anwendung der Regel $S \rightarrow A$, $j$-malige Anwendung von $A \rightarrow bAc$ und abschließende Anwendung von $A \rightarrow \lambda$ abgeleitet werden.\\
Damit gilt für jedes $w \in L_{1}$ demnach auch $w \in L(G_{1})$ und die Behauptung ist gezeigt.
\subsection{} %2.
\section{} %5.6
\subsection{} %1.
$z_{0}$ ist der Start- und $z_{4}$ der einzige Endzustand. Im Zustand $z_{0}$ können beliebig viele $a$s gelesen werden (auch keine). Anschließend kann bei leerer Eingabe in den Endzustand gewechselt werden oder bei einem $b$ in der Eingabe in den Zustand $z_{1}$. Der Zustand kann nur wieder verlassen werden, wenn mindestens ein weiteres $b$ von der Eingabe gelesen wird. Im Zustand $z_{2}$ befindet man sich demnach nur, wenn zuvor jeweils 2 $b$s gelesen wurden. Entweder direkt von $z_{0}$ nach $z_{1}$ und von $z_{1}$ nach $z_{2}$ oder von $z_{2}$ nach $z_{1}$ und von $z_{1}$ nach $z_{2}$.\\
Nur beim Wechsel von Zustand $z_{1}$ nach $z_{2}$ wird ein $B$ auf den Keller gelegt. Sind alle $b$s gelesen, liegen demnach also halb so viele $B$s auf dem Keller, wie $b$s gelesen wurden.\\
In den Zustand $z_{3}$ kann nur gewechselt werden, wenn ein $a$ gelesen wird und ein $B$ auf dem Keller liegt. $z_{3}$ kann erst wieder verlassen werden, wenn alle $B$s wieder vom Keller gelöscht worden sind.\\
Zum Schluss kann in den Endzustand gewechselt werden. Befindet man sich im Endzustand wurden also beliebig viele $a$s (auch keine), gefolgt von keinen weiteren Zeichen oder $2n$ $b$s und $n$ $a$s gelesen.\\
Ist $w \in L$, so enthält das Wort zunächst beliebig viele $a$s, dann keine weiteren Zeichen oder aber $2n$ $b$s und $n$ $a$s. Dies ist gleichbedeutend mit dem Endzustand des Automaten.\\
Ist umgekehrt $w \in L(A)$, so muss das Lesen von $w$ den Automaten $A$ in den Zustand $z_{4}$ überführen, da dies der einzige Endzustand ist. Dies ist aber gleichbedeutend damit, dass nach Konstruktion von $A$ auch $w = a^{m}b^{2n}a^{n}$, also $w \in L$ gilt.
\subsection{} %2.
\end{document}

99
fgi1/Aufgabenblatt6.tex Normal file
View File

@ -0,0 +1,99 @@
\documentclass[10pt,a4paper,oneside,ngerman,numbers=noenddot]{scrartcl}
\usepackage[T1]{fontenc}
\usepackage[utf8]{inputenc}
\usepackage[ngerman]{babel}
\usepackage{amsmath}
\usepackage{amsfonts}
\usepackage{amssymb}
\usepackage{paralist}
\usepackage{gauss}
\usepackage{stmaryrd}
\usepackage[locale=DE,exponent-product=\cdot,detect-all]{siunitx}
\usepackage{tikz}
\usetikzlibrary{automata,matrix,fadings,calc,positioning,decorations.pathreplacing,arrows,decorations.markings}
\usepackage{polynom}
\polyset{style=C, div=:,vars=x}
\pagenumbering{arabic}
\def\thesection{6.\arabic{section})}
\def\thesubsection{\arabic{subsection}.}
\def\thesubsubsection{(\roman{subsubsection})}
\setcounter{section}{3}
\makeatletter
\renewcommand*\env@matrix[1][*\c@MaxMatrixCols c]{%
\hskip -\arraycolsep
\let\@ifnextchar\new@ifnextchar
\array{#1}}
\makeatother
\addtolength{\parskip}{\baselineskip}
\begin{document}
\author{Jim Martens}
\title{Hausaufgaben zum 14. Mai}
\maketitle
\section{} %6.4
\subsection{} %1.
Zunächst werden links und rechts vom Wort Marker geschrieben. Der Lesekopf wird auf das letzte $a$ bewegt. Nun wird der Lesekopf nach links bewegt, bis der linke Marker erreicht wird. Auf dem Weg wird das erste, dritte, fünfte usw. gelesene a durch ein + ersetzt. Anschließend wird das Band noch einmal komplett durchlaufen, dabei werden die mit + beschriebenen Felder ignoriert. Die TM merkt sich die Parität ($0$, $1$, $0$, etc.) in jeweils einem Zustand. Das zuerst gelesene $a$ erzeugt die Parität $1$, das zweite $0$, usw. bis der Lesekopf den rechten Marker erreicht. Die aktuelle Parität wird auf das erste unbeschriebene Feld links vom linken Marker geschrieben.
Nun bewegt sich der Lesekopf zurück auf das rechte Ende des Wortes (letztes $a$). Jetzt bewegt sich der Lesekopf wieder nach links und ersetzt das erste, dritte, fünfte usw. gelesene $a$ durch ein +. Es wird wieder das Band abgelaufen und die Parität gemerkt. Das Ergebnis wird links vom linken Marker auf das erste unbeschriebene Feld geschrieben.
Dies wiederholt sich nun so lange bis zwischen den beiden Markern keine $a$s mehr stehen. Nun werden die ganzen +-Zeichen gelöscht und abschließend auch die Marker. Der Lesekopf bewegt sich auf den Anfang der jetzt noch beschriebenen Felder.
\subsection{} %2.
\begin{tikzpicture}[shorten >=1pt,node distance=2.0cm,on grid,auto,/tikz/initial text=,>=stealth',text centered]
\node[initial,state] (z0) {$z_{0}$};
\node[state] (z1) [below=2.0 of z0] {$z_{1}$};
\node[state] (z2) [right=2.0 of z1] {$z_{2}$};
\node[state] (z3) [right=3.0 of z0] {$z_{3}$};
\node[state] (z4) [below right=4.0 and 3.0 of z0] {$z_{4}$};
\node[state] (z5) [right=4.0 of z3] {$z_{5}$};
\node[state] (z6) [right=4.0 of z4] {$z_{6}$};
\node[state] (z7) [below right=2.0 and 2.0 of z5] {$z_{7}$};
\node[state] (z8) [left=2.0 of z7] {$z_{8}$};
\node[state] (z9) [below right=2.0 and 4.0 of z8] {$z_{9}$};
\node[state,accepting] (z10) [below=2.0 of z9] {$z_{10}$};
\path[every node/.style={font=\scriptsize}]
(z0) edge[loop above,->] node[above,align=center] {0,0,L\\1,1,L} (z0)
(z0) edge[->] node[left] {\#,\$,R} (z1)
(z1) edge[loop below,->] node[below,align=center] {0,0,R\\1,1R} (z1)
(z1) edge[->] node[above] {\#,\$,L} (z2)
(z2) edge[->] node[near start] {1,\#,R} (z3)
(z2) edge[->] node[below left] {0,\#,R} (z4)
(z3) edge[loop above,->] node[above] {\#,\#,R} (z3)
(z3) edge[->] node[near start] {\$,\$,R} (z5)
(z4) edge[loop below,->] node[below] {\#,\#,R} (z4)
(z4) edge[->] node[below] {\$,\$,R} (z6)
(z5) edge[loop above,->] node[above,align=center] {0,0,R\\1,1,R} (z5)
(z6) edge[loop below,->] node[below,align=center] {1,1,R\\0,0,R} (z6)
(z5) edge[->] node[near start] {\#,1,L} (z7)
(z6) edge[->] node[above left,near start] {\#,0,L} (z7)
(z2) edge[->] node[near start] {\$,\$,-} (z8)
(z7) edge[->] node[above] {\$,\$,L} (z8)
(z7) edge[loop right,->] node[right,align=center] {1,1,L\\0,0,L} (z7)
(z8) edge[->] node[near end] {1,\#,R} (z3)
(z8) edge[->] node[above] {0,\#,R} (z4)
(z8) edge[loop above,->] node[above] {\#,\#,L} (z8)
(z8) edge[->] node[above right,near end] {\$,\#,R}
(z9)
(z9) edge[loop right,->] node[right] {\#,\#,R} (z9)
(z9) edge[->] node[right] {\$,\#,R} (z10);
\end{tikzpicture}\\
Zu Beginn wird im Zustand $z_{0}$ solange nach links gegangen, bis man an den unbeschriebenen Rand kommt. Dann wird das erste unbeschriebene Feld durch ein \$-Zeichen ersetzt und im Zustand $z_{1}$ wird einmal bis an das rechte Ende des Wortes gelaufen. Auch dort wird im Übergang zu $z_{2}$ das erste unbeschriebene Feld durch \$-Zeichen ersetzt.
Damit steht der Lesekopf nun auf dem letzten Zeichen des Wortes $w$ und das Wort $w$ ist vorne und hinten durch ein \$-Zeichen begrenzt.
Anschließend wird entweder eine $0$ oder eine $1$ gelesen. Im Falle der $1$ wird sie durch $\#$ ersetzt. Der Lesekopf bewegt sich am rechten Marker vorbei und schreibt die $1$ auf das Band ($z_{2} \rightarrow z_{3} \rightarrow z_{5} \rightarrow z_{7}$. Nun bewegt er sich wieder am Marker vorbei ($z_{7} \rightarrow z_{8}$), durchläuft die unbeschriebenen Felder bis er auf eine $0$ oder eine $1$ trifft. Im Falle der $1$ löscht er diese und durchläuft alle unbeschriebenen Felder, die rechts davon stehen bis er wieder auf den rechten Marker trifft. Er durchläuft alle hier bereits geschriebenen Felder und schreibt auf das erste unbeschriebene Feld eine $1$.
Folgend werden alle bereits beschriebenen Felder und der Marker, sowie alle unbeschriebenen Felder links vom Marker durchlaufen, bis wieder auf eine $1$ oder eine $0$ getroffen wird. Für die $0$ funktioniert es analog. nur werden statt $z_{3}$ und $z_{5}$ die Zustände $z_{4}$ und $z_{6}$ durchlaufen.
Das wiederholt sich so lange, bis das ganze Wort gelesen wurde, zwischen den beiden Markern also nur noch unbeschriebene Felder stehen, und der Lesekopf am linken Marker ankommt.
Damit wurde das Wort $w$ nun komplett gelesen und in umgekehrter Reihenfolge rechts vom rechten Marker wieder aufgeschrieben.
Abschließend wird der linke Marker gelöscht, dann werden alle unbeschriebenen Felder nach rechts durchlaufen bis der Lesekopf auf den rechten Marker trifft. Dieser wird auch noch gelöscht und der Lesekopf steht am Beginn des umgekehrten Wortes $w$. Die TM ist im Endzustand und akzeptiert damit das Wort $w$.
Ist das Eingabewort leer, so wird direkt der linke Marker geschrieben ($z_{0} \rightarrow z_{1}$) und dann der rechte ($z_{1} \rightarrow z_{2}$), wobei der Lesekopf nun auf dem linken Marker steht. Er hält auf dem Marker ($z_{2} \rightarrow z_{8}$), löscht ihn und wechselt dabei auf den rechten Marker ($z_{8} \rightarrow z_{9}$), um auch diesen sofort zu löschen ($z_{9} \rightarrow z_{10}$). Der Lesekopf befindet sich nun zwei Felder weiter rechts als am Anfang, da aber das ganze Band nun genauso leer ist wie am Anfang, ist die Position irrelevant. Die TM ist auch hier im Endzustand, womit klar ist, dass das leere Wort das leere Wort ergibt.
\pagebreak
\section{} %6.5
\end{document}

61
fgi1/Aufgabenblatt7.tex Normal file
View File

@ -0,0 +1,61 @@
\documentclass[10pt,a4paper,oneside,ngerman,numbers=noenddot]{scrartcl}
\usepackage[T1]{fontenc}
\usepackage[utf8]{inputenc}
\usepackage[ngerman]{babel}
\usepackage{amsmath}
\usepackage{amsfonts}
\usepackage{amssymb}
\usepackage{paralist}
\usepackage{gauss}
\usepackage{stmaryrd}
\usepackage[locale=DE,exponent-product=\cdot,detect-all]{siunitx}
\usepackage{tikz}
\usetikzlibrary{automata,matrix,fadings,calc,positioning,decorations.pathreplacing,arrows,decorations.markings}
\usepackage{polynom}
\polyset{style=C, div=:,vars=x}
\pagenumbering{arabic}
\def\thesection{7.\arabic{section})}
\def\thesubsection{\arabic{subsection}.}
\def\thesubsubsection{(\roman{subsubsection})}
\setcounter{section}{2}
\makeatletter
\renewcommand*\env@matrix[1][*\c@MaxMatrixCols c]{%
\hskip -\arraycolsep
\let\@ifnextchar\new@ifnextchar
\array{#1}}
\makeatother
\addtolength{\parskip}{\baselineskip}
\begin{document}
\author{Jim Martens}
\title{Hausaufgaben zum 28. Mai}
\maketitle
\section{} %7.3
\subsection{} %1.
Ich erstelle zunächst ein Array $A$ aus der Länge $|V-1|$. In dieses Array sollen die Knotenbezeichner geschrieben werden. Die Reihenfolge der Knotenbezeichner von vorne nach hinten entspricht der Reihenfolge, in denen die Knoten durchlaufen werden. \\
Wenn es eine Verbindung von $s$ nach $t$ gibt, dann verbraucht sie maximal $O(V-1)$ Platz. Der Startknoten $s$ muss nicht gespeichert werden.\\
\\
Dies folgt so: Nichtdeterministisch wird für $A[1]$ ein Bezeichner der Nachfolgeknoten von $s$ gewählt. Anschließend wird nichtdeterministisch ein Wert für $A[2]$ ausgewählt, der ein Knotenbezeichner des Knoten sein muss, der für $A[1]$ gewählt wurde. Analog funktioniert es für $A[3]$, $A[4]$ usw. Um diese nichtdeterministischen Schritte zu verdeutlichen, gehe ich beispielsweise von $|V|=4$ (vier Knoten) und einem Array mit drei Elementen aus. $s$ sei $v_{1}$ und $t$ sei $v_{4}$. Ich gehe ferner von einer TM aus. So bin ich am Anfang dann in einem Zustand $z_{0}$ und der Arrayinhalt lässt sich darstellen als $[\#,\#,\#]$, wobei die $\#$ dafür stehen, dass das Arrayelement an dieser Stelle noch nicht initialisiert wurde. Nach dem ersten nichtdeterministischen Schritt sind wir nun im Zustand $z_{1}$ und der Arrayinhalt hat die Form $[v_{2},\#,\#]$ oder $[v_{3},\#,\#]$. Notiere ich die Konfiguration der TM als ein Tupel bestehend aus dem Zustand $z$ und dem Arrayinhalt, so bin ich nun also nichtdeterministisch in den Konfigurationen $(z_{1},[v_{2},\#,\#])$ und $(z_{1},[v_{3},\#,\#])$. Nun wird in den Zustand $z_{2}$ gewechselt und der Nachfolgeknoten für die bisher gewählten Knoten gewählt bzw. geraten. Ich bin nun in den Konfigurationen $(z_{2},[v_{2},v_{3},\#,\#])$, $(z_{2},[v_{2},v_{4},\#,\#])$, $(z_{2},[v_{3},v_{1},\#,\#])$ und $(z_{2},[v_{3},v_{2},\#,\#])$. Dies setzt sich nun fort, sodass ich dann am Ende nichtdeterministisch in den Konfigurationen $(z_{4},[x,y,z,w])$ bin, wobei $x,y,z$ und $w$ alle Nachfolgeknoten des jeweils vorherigen Knoten durchlaufen. Dies ist ähnlich zu einem NFA, der nach Lesen eines Symbols nichtdeterministisch in verschiedenen Zuständen sein kann.\\
\\
Zurück zu eigentlichen Aufgabe: Nachdem nun alle möglichen Pfade im Graphen nichtdeterministisch durchlaufen worden sind, braucht nur noch überprüft zu werden, ob der Pfad korrekt ist. Dazu muss jedes Array nur mittels for-each-Schleife durchlaufen werden und auf Vorhandensein von $t$ geprüft werden. Kommt $t$ im Array vor, dann ist dies eine Erfolgsrechnung und die Überprüfung weiterer Arrays ist irrelevant. Kommt $t$ in allen Arrays nicht vor, dann existiert kein Pfad von $s$ nach $t$.\\
\\
Die Laufzeit liegt bei maximal $O(2 \cdot (V-1))$, da zunächst maximal $O(V-1)$ Schritte benötigt werden, um den Pfad zu raten (man beachte, dass hier in einer Rechnung nur maximal $O(V-1)$ Schritte gemacht werden, auch wenn die Anzahl der Schritte in allen Rechnungen viel höher ist, aber es geht eben gerade um die (maximale) Anzahl der Schritte in einer Rechnung) und dann noch weitere maximal $O(V-1)$ Schritte, um den Pfad zu überprüfen. Der Platzbedarf liegt lediglich bei maximal $O(V-1)$, da dies ausreicht, um einen geratenen Pfad zu speichern. Auch hier beachte man, dass es nur um den (maximalen) Platzbedarf in einer Rechnung geht, nicht um den Platzbedarf über alle nichtdeterministischen Rechnungen.
\subsection{} %2.
Bei maximal logarithmisch viel Platz ist das Verfahren nahezu analog. Allerdings werden nicht alle Knoten eines Pfades (außer s) gespeichert, sondern immer nur der letzte.
Nehme ich wieder das Beispiel mit den vier Knoten, dann ergibt sich für $z_{0}$ $[v_{1}]$. Für $z_{1}$ ergibt sich $[v_{2}],[v_{3}]$. Für $z_{2}$ ergibt sich $[v_{1}],[v_{2}],[v_{3}],[v_{4}]$. In diesem Fall kann die TM bereits hier abbrechen, da bei den vier geratenen Knoten jeder Knoten enthalten ist. Weitere Schritte würden also keine zusätzlichen Infos bringen. Nun können die Rechnungen überprüft werden, wobei pro Rechnung maximal $O(V-1)$ Schritte für das Raten des Pfades und $O(1)$ Schritte für das Überprüfen nötig sind. Insgesamt benötigt der Algorithmus also $O(V)$ Schritte Zeit. Er verbraucht nur $O(1)$ Platz, was einem logarithmischen Platzverbrauch entspricht.\\
Für den Fall $|V|=2$ müsste gar nichts gespeichert werden, da die TM hier nur zwei Optionen hat: $s$ hat einen Nachfolgeknoten oder keinen. Gibt es einen, dann existiert eine Verbindung, gibt es keinen, dann nicht.
\section{} %7.4
\subsection{} %1.
Es werden nichtdeterministisch alle Kanten aus $E_{2}$ folgendermaßen überprüft:\\
\begin{enumerate}
\item Bestimmen der beiden verbundenen Knoten aus $V_{2}$.
\item Bestimmen der Bilder dieser Knoten
\item Nichtdeterministisch überprüfen, ob eine entsprechende Kante in $E_{1}$ existiert.
\item Gibt es keine solche Kante, dann kann abgebrochen werden.
\end{enumerate}
Wurde nicht abgebrochen, dann kann es Teilmengen geben, die die Bedingung erfüllen.
Der Algorithmus dauert $O(3)$ lang. Damit dauert der Algorithmus nur polynomiell lang.
\subsection{} %2.
\end{document}

137
fgi1/Aufgabenblatt8.tex Normal file
View File

@ -0,0 +1,137 @@
\documentclass[10pt,a4paper,oneside,ngerman,numbers=noenddot]{scrartcl}
\usepackage[T1]{fontenc}
\usepackage[utf8]{inputenc}
\usepackage[ngerman]{babel}
\usepackage{amsmath}
\usepackage{amsfonts}
\usepackage{amssymb}
\usepackage{paralist}
\usepackage{gauss}
\usepackage{stmaryrd}
\usepackage[locale=DE,exponent-product=\cdot,detect-all]{siunitx}
\usepackage{tikz}
\usetikzlibrary{automata,matrix,fadings,calc,positioning,decorations.pathreplacing,arrows,decorations.markings}
\usepackage{polynom}
\polyset{style=C, div=:,vars=x}
\pagenumbering{arabic}
\def\thesection{8.\arabic{section})}
\def\thesubsection{\arabic{subsection}.}
\def\thesubsubsection{(\roman{subsubsection})}
\setcounter{section}{1}
\makeatletter
\renewcommand*\env@matrix[1][*\c@MaxMatrixCols c]{%
\hskip -\arraycolsep
\let\@ifnextchar\new@ifnextchar
\array{#1}}
\makeatother
\addtolength{\parskip}{\baselineskip}
\begin{document}
\author{Jim Martens}
\title{Hausaufgaben zum 4. Juni}
\maketitle
\section{} %8.2
\textit{Behauptung}\\
Für alle Formeln F$\, \in \mathcal{L}_{AL}$ gilt, $|\text{Tf(F)}| \leq |\text{F}|$.\\
\\
\textit{Induktionsanfang}\\
Teilbeweis für die auf atomare Formeln eingeschränkte Behauptung: Für jedes Aussagensymbol
A$\, \in \mathcal{A}s_{AL}$ gilt: $|\text{Tf(A)}| \leq |\text{A}|$.\\
A hat nur eine Teilformel und zwar sich selbst. Die Länge von A beträgt ebenso eins. Demnach ergibt sich $1 \leq 1$, was offensichtlich gilt.\\
\\
\textit{Induktionsannahme}\\
Es seien G$_{1}, \,$G$_{2} \in \mathcal{L}_{AL}$ Formeln, für die gilt: $|\text{Tf(G}_{1}\text{)}| \leq |\text{G}_{1}|$ und $|\text{Tf(G}_{2}\text{)}| \leq |\text{G}_{2}|$.\\
\\
\textit{Induktionsschritt}\\
Fall: $\neg \,$G$_{1}$\\
Teilbeweis für $|\text{Tf(}\neg \text{G}_{1}\text{)}| \leq |\neg \,\text{G}_{1}|$.\\
Es gilt: $|\text{Tf(}\neg \,\text{G}_{1}\text{)}| = 1 + |\text{Tf(G}_{1}\text{)}| \overset{IA}{\leq} 1 + |\text{G}_{1}| = |\neg \,\text{G}_{1}|$\\
Demnach gilt die Behauptung für $\neg \,$G$_{1}$.\\
\\
Fall: (G$_{1} \circ \,$G$_{2}$) für $\circ \in \{\vee, \wedge, \Rightarrow, \Leftrightarrow\}$\\
Teilbeweis für $|\text{Tf(G}_{1} \circ \, \text{G}_{2}\text{)}| \leq |(\text{G}_{1} \circ \,\text{G}_{2})|$.\\
Es gilt:\\
\begin{alignat*}{2}
|\text{Tf(G}_{1} \circ \, \text{G}_{2}\text{)}| &=& 1 + |\text{Tf(G}_{1}\text{)}| + |\text{Tf(G}_{2}\text{)}| \\
1 + |\text{Tf(G}_{1}\text{)}| + |\text{Tf(G}_{2}\text{)}| &\overset{IA}{\leq}& 1 + |\text{G}_{1}| + |\text{G}_{2}| \\
1 + |\text{G}_{1}| + |\text{G}_{2}| &\leq & 3 + |\text{G}_{1}| + |\text{G}_{2}| \\
3 + |\text{G}_{1}| + |\text{G}_{2}| &=& |(\text{G}_{1} \circ \,\text{G}_{2})|
\end{alignat*}
Demnach gilt die Behauptung für (G$_{1} \circ \,$G$_{2}$).\\
\\
\textit{Resumé}\\
Nach dem Prinzip der strukturellen Induktion ergibt sich damit: Für alle Formeln F$\, \in \mathcal{L}_{AL}$
gilt, $|\text{Tf(F)}| \leq |\text{F}|$.
%
%
%
\section{} %8.3
\textit{Behauptung}\\
Für alle Formeln F$\, \in \mathcal{L}_{AL}$ gilt, $|$F$| \leq 2^{|\text{Tf(F)}|+1}-3$.\\
\\
\textit{Induktionsanfang}\\
Teilbeweis für die auf atomare Formeln eingeschränkte Behauptung: Für jedes Aussagensymbol
A$\, \in \mathcal{A}s_{AL}$ gilt: $|$A$| \leq 2^{|\text{Tf(A)}|+1}-3$.\\
Die Länge von A beträgt 1. Ebenso hat A lediglich eine Teilformel und zwar sich selbst. Daraus ergibt sich:\\
\begin{alignat*}{2}
1 &\leq & 2^{1 + 1}-3 \\
1 &\leq & 2^{2} - 3 \\
1 &\leq & 4-3 = 1
\end{alignat*}
Dies gilt offensichtlich.\\
\\
\textit{Induktionsannahme}\\
Es seien G$_{1}, \,$G$_{2} \in \mathcal{L}_{AL}$ Formeln, für die gilt: $|$G$_{1}| \leq 2^{|\text{Tf(G}_{1}\text{)}|+1}-3$ und $|$G$_{2}| \leq 2^{|\text{Tf(G}_{2}\text{)}|+1}-3$.\\
\\
\textit{Induktionsschritt}\\
Fall: $\neg \,$G$_{1}$\\
Teilbeweis für $|\neg \,$G$_{1}| \leq 2^{|\text{Tf(}\neg \,\text{G}_{1}\text{)}|+1}-3$.\\
Es gilt:\\
\begin{alignat*}{2}
|\neg \,\text{G}_{1}| = 1 + |\text{G}_{1}| &\overset{IA}{\leq} & 1 + 2^{|\text{Tf(G}_{1}\text{)}|+1}-3 \\
1 + 2^{|\text{Tf(G}_{1}\text{)}|+1}-3 &\leq & \left(2^{|\text{Tf(G}_{1}\text{)}|+1}-3\right) + \left(2^{|\text{Tf(G}_{1}\text{)}|+1}-3\right) \\
\left(2^{|\text{Tf(G}_{1}\text{)}|+1}-3\right) + \left(2^{|\text{Tf(G}_{1}\text{)}|+1}-3\right) &=&
2\cdot 2^{|\text{Tf(G}_{1}\text{)}|+1}-3 \\
&=& 2^{|\text{Tf(G}_{1}\text{)}|+1+1}-3 \\
&=& 2^{|\text{Tf(}\neg \,\text{G}_{1}\text{)}|+1}-3
\end{alignat*}
Die Behauptung gilt demnach für $\neg \,$G$_{1}$.\\
\\
Fall: (G$_{1} \circ \,$G$_{2}$) für $\circ \in \{\vee, \wedge, \Rightarrow, \Leftrightarrow\}$\\
Teilbeweis für $|($G$_{1} \circ \,$G$_{2})| \leq 2^{|\text{Tf(G}_{1} \circ \,\text{G}_{2}\text{)}|+1}-3$ .\\
Es gilt:\\
\begin{alignat*}{2}
|\text{G}_{1} \circ \,\text{G}_{2}| = 3 + |\text{G}_{1}| + |\text{G}_{2}| &\overset{IA}{\leq}& 3 + 2^{|\text{Tf(G}_{1}\text{)}|+1}-3 + 2^{|\text{Tf(G}_{2}\text{)}|+1}-3 \\
3 + 2^{|\text{Tf(G}_{1}\text{)}|+1}-3 + 2^{|\text{Tf(G}_{2}\text{)}|+1}-3 &=& 2^{|\text{Tf(G}_{1}\text{)}|+1} + 2^{|\text{Tf(G}_{2}\text{)}|+1}-3 \\
2^{|\text{Tf(G}_{1}\text{)}|+1} + 2^{|\text{Tf(G}_{2}\text{)}|+1}-3 &=& 2 \cdot \left(2^{|\text{Tf(G}_{1}\text{)}|}+2^{|\text{Tf(G}_{2}\text{)}|}\right) -3 \\
2 \cdot \left(2^{|\text{Tf(G}_{1}\text{)}|}+2^{|\text{Tf(G}_{2}\text{)}|}\right) -3 &\leq & 2 \cdot \left(2^{|\text{Tf(G}_{1}\text{)}|} \cdot 2^{|\text{Tf(G}_{2}\text{)}|}\right) -3 \\
2 \cdot \left(2^{|\text{Tf(G}_{1}\text{)}|} \cdot 2^{|\text{Tf(G}_{2}\text{)}|}\right) -3 &=& 2 \cdot 2^{|\text{Tf(G}_{1}\text{)}|+|\text{Tf(G}_{2}\text{)}|} -3 \\
2 \cdot 2^{|\text{Tf(G}_{1}\text{)}|+|\text{Tf(G}_{2}\text{)}|} -3 &\leq & 2 \cdot 2^{|\text{Tf(G}_{1}\text{)}|+|\text{Tf(G}_{2}\text{)}| + 1} -3 \\
2 \cdot 2^{|\text{Tf(G}_{1}\text{)}|+|\text{Tf(G}_{2}\text{)}| + 1} -3 &=& 2 \cdot 2^{|\text{Tf(G}_{1} \circ \,\text{G}_{2}\text{)}|} -3 \\
&=& 2^{|\text{Tf(G}_{1} \circ \,\text{G}_{2}\text{)}|+1} - 3
\end{alignat*}
Demnach gilt die Behauptung für (G$_{1} \circ \,$G$_{2}$).\\
\\
\textit{Resumé}\\
Nach dem Prinzip der strukturellen Induktion ergibt sich damit: Für alle Formeln F$\, \in \mathcal{L}_{AL}$
gilt, $|$F$| \leq 2^{|\text{Tf(F)}|+1}-3$.
\section{} %8.4
\subsection{} %1.
\begin{alignat*}{2}
F^{n}_{a} &=& \begin{cases}
A & n=1\\
\neg A & n=2 \\
\neg F^{(n-1)}_{a} & n > 2
\end{cases}
\end{alignat*}
\subsection{} %2.
\begin{alignat*}{2}
F^{n}_{b} &=& \begin{cases}
A & n=1 \\
\neg A & n=2 \\
(A \wedge B) & n=3 \\
\neg F^{(n-1)}_{b} & n > 3
\end{cases}
\end{alignat*}
\end{document}

187
fgi1/Aufgabenblatt9.tex Normal file
View File

@ -0,0 +1,187 @@
\documentclass[10pt,a4paper,oneside,ngerman,numbers=noenddot]{scrartcl}
\usepackage[T1]{fontenc}
\usepackage[utf8]{inputenc}
\usepackage[ngerman]{babel}
\usepackage{amsmath}
\usepackage{amsfonts}
\usepackage{amssymb}
\usepackage{paralist}
\usepackage{gauss}
\usepackage{stmaryrd}
\usepackage[locale=DE,exponent-product=\cdot,detect-all]{siunitx}
\usepackage{tikz}
\usetikzlibrary{automata,matrix,fadings,calc,positioning,decorations.pathreplacing,arrows,decorations.markings}
\usepackage{polynom}
\polyset{style=C, div=:,vars=x}
\pagenumbering{arabic}
\def\thesection{9.\arabic{section})}
\def\thesubsection{\arabic{subsection}.}
\def\thesubsubsection{(\alph{subsubsection})}
\setcounter{section}{1}
\makeatletter
\renewcommand*\env@matrix[1][*\c@MaxMatrixCols c]{%
\hskip -\arraycolsep
\let\@ifnextchar\new@ifnextchar
\array{#1}}
\makeatother
\addtolength{\parskip}{\baselineskip}
\begin{document}
\author{Jim Martens}
\title{Hausaufgaben zum 11. Juni}
\maketitle
\section{} %9.2
\subsection{} %1.
\subsubsection{} %(a)
Anna wohnt in Aachen oder Bernd wohnt in Berlin oder Carl wohnt in Chemnitz.
\subsubsection{} %(b)
Bernd wohnt in Berlin oder Carl wohnt in Chemnitz oder Anna wohnt in Aachen.
\subsubsection{} %(c)
Wenn Anna in Aachen wohnt, dann wohnt Bernd in Berlin oder Carl wohnt in Chemnitz.
\subsubsection{} %(d)
Wenn Anna in Aachen wohnt und Bernd in Berlin wohnt, dann wohnt Carl in Chemnitz.
\subsubsection{} %(e)
Wenn Anna in Aachen wohnt, dann wohnt Carl in Chemnitz, wenn Bernd in Berlin wohnt.
\subsubsection{} %(v)
Bernd wohnt genau dann in Berlin, wenn Anna in Aachen wohnt und Carl in Chemnitz wohnt.
\subsection{} %2.
\begin{enumerate}
\item A$\, \Rightarrow \,$A
Die Formel ist allgemeingültig und erfüllbar (siehe 9.2 (a)).
\item A$\, \Rightarrow \,\neg$A
Die Formel ist erfüllbar, falsifizierbar und kontingent (siehe 9.2 (b)).
\item A$\, \Leftrightarrow \,\neg$A
Die Formel ist falsifizierbar und unerfüllbar (siehe 9.2 (c)).
\item A$\, \vee \,\neg$A
Die Formel ist allgemeingültig und erfüllbar. Egal welche Belegung A hat, eine der beiden Teilformeln (A, $\neg$A) ist immer wahr.
\item A$\, \wedge \,\neg$A
Die Formel ist unerfüllbar und falsifizierbar. Egal welche Belegung A hat, nur eine der beiden Teilformeln (A, $\neg$A) kann wahr sein.
\item A$\, \wedge \,$A
Die Formel ist erfüllbar, falsifizierbar und kontingent. Je nach Belegung ist die Formel entweder falsifiziert (falsch und falsch) oder erfüllt (wahr und wahr).
\item A$\, \vee \,$A
Die Formel ist erfüllbar, falsifizierbar und kontingent. Je nach Belegung ist die Formel entweder falsifiziert (falsch oder falsch) oder erfüllt (wahr oder wahr).
\item A$\, \Leftrightarrow \,$A
Die Formel ist allgemeingültig und erfüllbar. Je nach Belegung ergibt sich entweder (wahr biimpliziert wahr) oder (falsch biimpliziert falsch). In beiden Fällen ist die Formel erfüllt.
\item (A$\, \vee \,\neg$A)$\, \Rightarrow \,$(A$\, \wedge \neg$A)
Die Formel ist unerfüllbar und falsifizierbar. Die linke Teilformel wurde bereits als allgemeingültig gezeigt. Die rechte Teilformel wurde bereits als unerfüllbar gezeigt. Damit ist auch die ganze Formel unerfüllbar.
\item (A$\, \wedge \neg$A)$\, \Rightarrow \,$(A$\, \vee \,\neg$A)
Die Formel ist allgemeingültig und erfüllbar. Die linke Teilformel wurde bereits als unerfüllbar gezeigt und die rechte Teilformel wurde bereits als allgemeingültig gezeigt. Damit ist auch die ganze Formel allgemeingültig.
\item A$\, \Leftarrow \,\neg$A
Die Formel ist erfüllbar, falsifizierbar und kontingent. Je nach Belegung ergibt sich falsifiziert (wahr impliziert falsch) oder erfüllt (falsch impliziert wahr). Da die Formel kontingent ist, kann sie nicht allgemeingültig oder unerfüllbar sein.
\item (A$\, \Leftrightarrow \,$A)$\, \Rightarrow \,$(A$\, \Leftrightarrow \,\neg$A)
Die Formel ist unerfüllbar und falsifizierbar. Die linke Teilformel wurde bereits als allgemeingültig gezeigt. Die rechte Teilformel wurde bereits als unerfüllbar gezeigt. Damit ist auch die ganze Formel unerfüllbar.
\end{enumerate}
\subsection{} %3.
\subsubsection{} %(a)
Die Formel kann erfüllbar, allgemeingültig und eine Tautologie sein. Sie kann nicht falsifizierbar, unerfüllbar, kontingent und damit auch keine Kontradiktion sein.
Da T allgemeingültig ist, ist somit die Formel C $\, \Rightarrow \,$T immer erfüllt und demnach ebenso allgemeingültig. Da sie allgemeingültig ist, ist sie eine Tautologie, erfüllbar, nicht falsifizierbar, nicht kontingent, nicht unerfüllbar und keine Kontradiktion.
\subsubsection{} %(b)
Die Formel kann erfüllbar, falsifizierbar, allgemeingültig, eine Tautologie und kontingent sein. Sie kann nicht unerfüllbar und damit auch keine Kontradiktion sein.
Da T allgemeingültig ist, hängt der Wahrheitswert der Formel von E ab. Da E erfüllbar ist, ist die Formel auf jeden Fall nicht unerfüllbar und damit auch keine Kontradiktion. Allerdings kann E auch falsifizierbar und damit kontingent oder allgemeingültig und eine Tautologie sein.
\subsubsection{} %(c)
Die Formel kann allgemeingültig, erfüllbar und eine Tautologie sein. Sie kann nicht falsifizierbar, unerfüllbar oder kontingent sein und ist damit auch keine Kontradiktion.
K und U sind beide unerfüllbar und eine Kontradiktion, da diese beiden Eigenschaften äquivalent sind. Da beide Formeln somit unabhängig von der Belegung immer falsch ergeben, ist die Aussage K$\, \Leftrightarrow \,$ U wiederum immer richtig, womit die Formel erfüllbar, allgemeingültig und damit eine Tautologie ist. Damit ist die Formel nicht unerfüllbar, keine Kontradiktion, nicht falsifizierbar und nicht kontingent.
\subsubsection{} %(d)
Die Formel kann erfüllbar, falsifizierbar und kontingent sein. Sie kann nicht allgemeingültig, unerfüllbar und damit auch keine Tautologie oder Kontradiktion sein.
Da A allgemeingültig ist, hängt der Wahrheitswert von C ab. Da C kontingent ist, kann die Formel sowohl erfüllt als auch falsifiziert werden. Wenn C durch eine Belegung falsifiziert wird, dann auch die Formel A$\, \Rightarrow \,$C und wenn C erfüllt wird, dann auch die Formel A$\, \Rightarrow \,$C.
Somit ist die Formel weder allgemeingültig und eine Tautologie noch unerfüllbar und eine Kontradiktion.
\subsubsection{} %(e)
Die Formel kann falsifizierbar, unerfüllbar und damit eine Kontradiktion sein. Sie kann nicht erfüllbar, kontingent, allgemeingültig und damit auch keine Tautologie sein.
Da U unerfüllbar ist, wird die Formel immer falsifiziert. Daher ist die Formel falsifizierbar, unerfüllbar und eine Kontradiktion. Deswegen ist sie nicht erfüllbar, kontingent, allgemeingültig und damit auch keine Tautologie.
\subsubsection{} %(f)
Die Formel kann erfüllbar, falsifizierbar, kontingent, unerfüllbar und eine Kontradiktion sein. Sie kann nicht allgemeingültig und damit eine Tautologie sein.
Da U unerfüllbar ist, hängt der Wahrheitswert von E ab. Ist E erfüllt, ist die Formel E$\, \Rightarrow \,$U falsifiziert. Wenn E allgemeingültig ist, dann ist die Formel unerfüllbar. Ist E hingegen kontingent, dann ist die Formel auch erfüllbar und ebenso kontingent. Allgemeingültig kann die Formel jedoch auf keinen Fall sein.
\subsubsection{} %(g)
Die Formel kann erfüllbar, falsifizierbar, kontingent und allgemeingültig und damit eine Tautologie sein. Sie kann nicht unerfüllbar und eine Kontradiktion sein.
Da E und C beide erfüllbar sind, gibt es mindestens eine Belegung, bei der die Formel erfüllt ist. Wenn E allgemeingültig ist, dann ist auch die Formel allgemeingültig. Wenn E kontingent ist, dann ist die Formel falsifizierbar und damit selber kontingent.
Unerfüllbar ist die Formel nicht.
\section{} %9.3
\subsection{} %1.
\begin{tabular}{lr}
A$\, \Leftrightarrow \,$B & Elimination $\Leftrightarrow$ \\
$\equiv ($A$\, \Rightarrow \,$B$) \wedge ($B$ \, \Rightarrow \,$A$)$ & Elimination $\Rightarrow$ \\
$\equiv (\neg$A$\, \vee \,$B$) \wedge (\neg$B$\, \vee \,$A$)$ & KNF \\
A$\, \Leftrightarrow \,$B & Elimination $\Leftrightarrow$ \\
$\equiv ($A$\, \wedge \,$B$) \vee (\neg$A$\, \wedge \neg$B$)$ & DNF \\
A$\, \Leftrightarrow \,$C & Elimination $\Leftrightarrow$ \\
$\equiv ($A$\, \Rightarrow \,$C$) \wedge ($C$ \, \Rightarrow \,$A$)$ & Elimination $\Rightarrow$ \\
$\equiv (\neg$A$\, \vee \,$C$) \wedge (\neg$C$\, \vee \,$A$)$ & KNF \\
A$\, \Leftrightarrow \,$C & Elimination $\Leftrightarrow$ \\
$\equiv ($A$\, \wedge \,$C$) \vee (\neg$A$\, \wedge \neg$C$)$ & DNF \\
KNF-Erzeugung: \\
$($A$\, \Leftrightarrow \,$B$) \Rightarrow ($A$\, \Leftrightarrow \,$C$)$ & Elimination $\Rightarrow$ \\
$\equiv \neg(($A$\, \Leftrightarrow \,$B$) \wedge \neg($A$\, \Leftrightarrow \,$C$))$ \\
Ich bearbeite jetzt die beiden Teile der Konjunktion getrennt, \\
mit dem Ziel, für diese KNFen zu erzeugen:\\
$($A$\, \Leftrightarrow \,$B$)$ & Einsetzung der KNF \\
$\equiv (\neg$A$\, \vee \,$B$) \wedge (\neg$B$\, \vee \,$A$)$ & KNF\\
$\neg($A$\, \Leftrightarrow \,$C$)$ & Einsetzung der DNF \\
$\equiv \neg(($A$\, \wedge \,$C$) \vee (\neg$A$\, \wedge \neg$C$))$ & de Morgan, Doppelte Negation \\
$\equiv (\neg$A$\, \vee \neg$C$) \wedge ($A$\, \vee \,$C$)$ & KNF \\
Die Verknüpfung dieser beiden Teilresultate ergibt: \\
$\neg(($A$\, \Leftrightarrow \,$B$) \wedge \neg($A$\, \Leftrightarrow \,$C$))$ \\
$\neg((\neg$A$\, \vee \,$B$) \wedge (\neg$B$\, \vee \,$A$) \wedge (\neg$A$\, \vee \neg$C$) \wedge ($A$\, \vee \,$C$)) = \,$G & KNF \\
DNF-Erzeugung: \\
$($A$\, \Leftrightarrow \,$B$) \Rightarrow ($A$\, \Leftrightarrow \,$C$)$ & Elimination $\Rightarrow$ \\
$\equiv \neg($A$\, \Leftrightarrow \,$B$) \vee ($A$\, \Leftrightarrow \,$C$)$ \\
Ich bearbeite jetzt die beiden Teile der Disjunktion getrennt,\\
mit dem Ziel, für diese DNFen zu erzeugen:\\
$\neg($A$\, \Leftrightarrow \,$B$)$ & Einsetzung der KNF \\
$\equiv \neg((\neg$A$\, \vee \,$B$) \wedge (\neg$B$\, \vee \,$A$))$ & de Morgan, Doppelte Negation \\
$\equiv ($A$\, \wedge \neg$B$) \vee ($B$\, \wedge \, \neg$A$)$ & DNF \\
$($A$\, \Leftrightarrow \,$C$)$ & Einsetzung der DNF \\
$\equiv ($A$\, \wedge \,$C$) \vee (\neg$A$\, \wedge \neg$C$)$ & DNF \\
Die Verknüpfung dieser beiden Teilresultate ergibt: \\
$\neg($A$\, \Leftrightarrow \,$B$) \vee ($A$\, \Leftrightarrow \,$C$)$ \\
$\equiv ($A$\, \wedge \neg$B$) \vee ($B$\, \wedge \, \neg$A$) \vee ($A$\, \wedge \,$C$) \vee (\neg$A$\, \wedge \neg$C$) = \,$F & DNF
\end{tabular}
\subsection{} %2.
\begin{tabular}{c|ccc|ccc}
& A & B & C & (A$\, \Leftrightarrow \,$B) & (A$\, \Leftrightarrow \,$C) & ((A$\, \Leftrightarrow \,$B)$\, \Rightarrow \,$(A$\, \Leftrightarrow \,$C)) \\
\hline
$\mathcal{A}_{0}$ & 0 & 0 & 0 & 1 & 1 & 1 \\
$\mathcal{A}_{1}$ & 0 & 0 & 1 & 1 & 0 & 0 \\
$\mathcal{A}_{2}$ & 0 & 1 & 0 & 0 & 1 & 1 \\
$\mathcal{A}_{3}$ & 0 & 1 & 1 & 0 & 0 & 1 \\
$\mathcal{A}_{4}$ & 1 & 0 & 0 & 0 & 0 & 1 \\
$\mathcal{A}_{5}$ & 1 & 0 & 1 & 0 & 1 & 1 \\
$\mathcal{A}_{6}$ & 1 & 1 & 0 & 1 & 0 & 0 \\
$\mathcal{A}_{7}$ & 1 & 1 & 1 & 1 & 1 & 1
\end{tabular}\\
\\
\begin{tabular}{c|ccc|ccccc}
& A & B & C & $(\neg$A$\, \vee \,$B$)$ & $(\neg$B$\, \vee \,$A$)$ & $(\neg$A$\, \vee \neg$C$)$ & $($A$\, \vee \,$C$)$ & G \\
\hline
$\mathcal{A}_{0}$ & 0 & 0 & 0 & 1 & 1 & 1 & 0 & 1 \\
$\mathcal{A}_{1}$ & 0 & 0 & 1 & 1 & 1 & 1 & 1 & 0 \\
$\mathcal{A}_{2}$ & 0 & 1 & 0 & 1 & 0 & 1 & 0 & 1 \\
$\mathcal{A}_{3}$ & 0 & 1 & 1 & 1 & 0 & 1 & 1 & 1 \\
$\mathcal{A}_{4}$ & 1 & 0 & 0 & 0 & 1 & 1 & 1 & 1 \\
$\mathcal{A}_{5}$ & 1 & 0 & 1 & 0 & 1 & 0 & 1 & 1 \\
$\mathcal{A}_{6}$ & 1 & 1 & 0 & 1 & 1 & 1 & 1 & 0 \\
$\mathcal{A}_{7}$ & 1 & 1 & 1 & 1 & 1 & 0 & 1 & 1
\end{tabular}\\
\\
\begin{tabular}{c|ccc|ccccc}
& A & B & C & $($A$\, \wedge \neg$B$)$ & $($B$\, \wedge \, \neg$A$)$ & $($A$\, \wedge \,$C$)$ & $(\neg$A$\, \wedge \neg$C$)$ & F \\
\hline
$\mathcal{A}_{0}$ & 0 & 0 & 0 & 0 & 0 & 0 & 1 & 1 \\
$\mathcal{A}_{1}$ & 0 & 0 & 1 & 0 & 0 & 0 & 0 & 0 \\
$\mathcal{A}_{2}$ & 0 & 1 & 0 & 0 & 1 & 0 & 1 & 1 \\
$\mathcal{A}_{3}$ & 0 & 1 & 1 & 0 & 1 & 0 & 0 & 1 \\
$\mathcal{A}_{4}$ & 1 & 0 & 0 & 1 & 0 & 0 & 0 & 1 \\
$\mathcal{A}_{5}$ & 1 & 0 & 1 & 1 & 0 & 1 & 0 & 1 \\
$\mathcal{A}_{6}$ & 1 & 1 & 0 & 0 & 0 & 0 & 0 & 0 \\
$\mathcal{A}_{7}$ & 1 & 1 & 1 & 0 & 0 & 1 & 0 & 1
\end{tabular}\\
Der Wahrheitswert in den drei jeweils letzten Spalten ist identisch. Entsprechend sind die drei Formeln äquivalent.
\subsection{} %3.
\end{document}

25
fgi1/Template.tex Normal file
View File

@ -0,0 +1,25 @@
\textit{Behauptung}\\
Für alle Formeln F$\, \in \mathcal{L}_{AL}$ gilt, [Behauptung formuliert mit F].\\
\\
\textit{Induktionsanfang}\\
Teilbeweis für die auf atomare Formeln eingeschränkte Behauptung: Für jedes Aussagensymbol
A$\, \in \mathcal{A}s_{AL}$ gilt: [Behauptung formuliert mit A].\\
\\
\textit{Induktionsannahme}\\
Es seien G$_{1}, \,$G$_{2} \in \mathcal{L}_{AL}$ Formeln, für die gilt: [Behauptung formuliert mit G$_{1}$] und [Behauptung
formuliert mit G$_{2}$].\\
\\
\textit{Induktionsschritt}\\
Fall: $\neg \,$G$_{1}$\\
Teilbeweis für [Behauptung formuliert mit $\neg \,$G$_{1}$].\\
(Dieser Teilbeweis darf auf die Induktionsannahme zurückgreifen.)\\
\\
Fall: (G$_{1} \circ \,$G$_{2}$) für $\circ \in \{\vee, \wedge, \Rightarrow, \Leftrightarrow\}$\\
Teilbeweis für [Behauptung formuliert mit (G$_{1} \circ \,$G$_{2}$)].\\
(Dieser Teilbeweis darf auf die Induktionsannahme zurückgreifen. Dabei kann es sein, dass
man alle Operatoren gleich behandeln kann, oder man muss eine Fallunterscheidung nach
Operator machen. Dann kann es hier bis zu 4 Teilbeweise geben.)\\
\\
\textit{Resumé}\\
Nach dem Prinzip der strukturellen Induktion ergibt sich damit: Für alle Formeln F$\, \in \mathcal{L}_{AL}$
gilt, [Behauptung formuliert mit F].